Wikipedia:Reference desk/Science: Difference between revisions

From Wikipedia, the free encyclopedia
Content deleted Content added
Scsbot (talk | contribs)
edited by robot: adding date header(s)
 
Line 1: Line 1:
<!--- Please DO NOT enter your question at the top here. Put it at the bottom of the page. An easy way to do this is by clicking the "new section" tab ---><noinclude>{{Wikipedia:Reference desk/header|WP:RD/S}}
{{Unicode|}}
[[Category:Non-talk pages automatically signed by HagermanBot]]
[[Category:Non-talk pages that are automatically signed]]
[[Category:Pages monitored by bots|HagermanBot]]
[[Category:Pages automatically checked for incorrect links]]
[[Category:Wikipedia resources for researchers]]
{{Wikipedia:Reference desk/headercfg}}
[[Category:Wikipedia help forums]]
[[Category:Wikipedia reference desk|Science]]
[[Category:Wikipedia help pages with dated sections]] </noinclude>


= May 12 =
{{Wikipedia:Reference_desk/Archives/Science/2007 May 26}}


== ecological spray bottle ==


does anyone know if there are any glass or metal spray bottles with bioplastic triggers and straw available anywhere in existence? i really want to go plastic free for my succulent business ninosckasnaturals.com [[Special:Contributions/2600:1700:9758:7D90:B406:C016:3BC0:D48B|2600:1700:9758:7D90:B406:C016:3BC0:D48B]] ([[User talk:2600:1700:9758:7D90:B406:C016:3BC0:D48B|talk]]) 06:05, 12 May 2024 (UTC)
{{Wikipedia:Reference_desk/Archives/Science/2007 May 27}}
:Maybe one of those old-fashioned perfume misters with the rubber squeeze bulb? I doubt very much that there is a mass-produced non-plastic alternative spray bottle apparatus. <span style="font-family: Cambria;"> [[User:Abductive|<span style="color: teal;">'''Abductive'''</span>]] ([[User talk:Abductive|reasoning]])</span> 21:49, 12 May 2024 (UTC)
= May 28 =


:There are pump-type plant misters [https://m.media-amazon.com/images/I/51BjF5WYZIL._AC_UL320_.jpg (e.g. metal] or [https://m.media-amazon.com/images/I/61HSSmSKn2L._AC_UL320_.jpg glass]). --[[Special:Contributions/136.54.106.120|136.54.106.120]] ([[User talk:136.54.106.120|talk]]) 18:27, 13 May 2024 (UTC)
== What is the largest encyclopedia ever existed? ==
::P.s.: LOOPSEED sells stainless steel plant mister spray bottles in various finishes, well-suited for succulents (search online for details). --[[Special:Contributions/136.54.106.120|136.54.106.120]] ([[User talk:136.54.106.120|talk]]) 18:49, 13 May 2024 (UTC) <small>-- [edit: 22:33, 13 May 2024 (UTC)]</small>


== origin of the formula for LC frequency ==
I have read from a webpage (from wikipedia) that Wikipedia is larger than Britannica.
I would like to know if other (online, offline, like books) encyclopedias exists (or existed) who are larger than wikipedia.
Thank you.


In electricity, properties known as inductance and capacitance together can resonate. The formula for the frequency of resonance is 1/(2*Pi*SQRT(L*C)). Who first published this formula? ```` [[User:Dionne Court|Dionne Court]] ([[User talk:Dionne Court|talk]]) 06:33, 12 May 2024 (UTC)
:See [[Wikipedia:Size comparisons]], Wikipedia is one of the biggest encyclopaedia known to human kind, depending on how you see it, maybe the biggest. --[[User: Antilived|antilived]]<sup>[[User_talk:Antilived|T]] | [[Special:Contributions/Antilived|C]] | [[User:Antilived/Gallery|G]]</sup> 11:17, 28 May 2007 (UTC)


:Somewhat after Laplace 1800 and before Poincarre, 1899 with a strong suspicion that the ubiquitous Maxwell might have done it. [[User:Greglocock|Greglocock]] ([[User talk:Greglocock|talk]]) 06:57, 12 May 2024 (UTC)
::Though it will never be, nor aspire to be, as big as [[The Library of Babel]]. ;-) --[[User:140.247.240.18|140.247.240.18]] 14:10, 28 May 2007 (UTC)
:::Ironically, even with the larger article and word count, Wikipedia is readily available in electronic form and can thus be miniaturized to small volume (such as a few hard-hard-drives - I don't know how many terabytes it would take to save the whole encyclopedia, but this [[Wikipedia:Sound/list|collection]] of [[Public Domain]] classical music just set my own system back a couple of gigabytes....) [[User:Nimur|Nimur]] 15:03, 28 May 2007 (UTC)


::According to [[LC_circuit#History]] it was the ubiquitous [[Lord Kelvin]] in 1853. --[[User:Wrongfilter|Wrongfilter]] ([[User talk:Wrongfilter|talk]]) 07:10, 12 May 2024 (UTC)
::::The pages themselves are usually fairly small, it's the images that take a bunch of space. Still, even with all the images on commons and all the text on en., I imagine it would be very possible to store almost the entire encyclopedia on a single computer... actually, it sounds like a fun project :) go through the dumps and clean up articles and release them in a permenent "checked" version. Kinda like the Wikipedia CD, but online --<small> [[User:Feba|Phoeba Wright]]<sup>[[User_Talk:Feba|OBJECTION!]]</sup> </small> 16:07, 28 May 2007 (UTC)
:::Thank you. That man did everything. [[User:Greglocock|Greglocock]] ([[User talk:Greglocock|talk]]) 23:49, 12 May 2024 (UTC)
:::It does say that, but it is incorrect, which is why I posted here. Kelvin derived an equation to describe the transient response (response to a one-time shock excitation). However the article io [[LC_circuit#History]] gave as a reference an article in the Bell System Technical Journal, 1941, pages 415-453. I have now obtained this paper and it gives James Clerk Maxwell as the first to give the resonance formula (in a different but mathematically equivalent form), in a letter published in Philosphosical Magazine 1868. I will try and get this letter. ```` [[User:Dionne Court|Dionne Court]] ([[User talk:Dionne Court|talk]]) 03:13, 13 May 2024 (UTC)
::::It is [https://books.google.com/books?id=8nRLYOTfUN4C&pg=RA1-PA360&dq=%22On+Mr.+Grove's+Experiment+in+Magneto-electric+Induc-tion%22&hl=en this letter]. &nbsp;--[[User talk:Lambiam#top|Lambiam]] 12:14, 13 May 2024 (UTC)
:::::It's from the right guy and via the correct other guy, but it has no math in it at all. It is not therefore the earliest statement of the resonance formula.
:::::I'm looking for the fist statement of the formula as given in modern textbooks, i.e.,
:::::f = 1/(2π(LC)<sup>½</sup>).
:::::It is a trivial exercise in algrbra to convert Maxwell's form into the standard modern form, but I would like to know when the modern form was first give. [[User:Dionne Court|Dionne Court]] ([[User talk:Dionne Court|talk]]) 00:29, 14 May 2024 (UTC)
::::::The section entitled {{serif|''Mathematical Theory of the Experiment''}}, an enclosure to the letter immediately following it on page 361, definitely contains some maths. On page 363 we see the equation <math>\text{CL}n^2=1,</math> which results in an amplitude that, Maxwell writes, "{{serif|is the greatest effect which can be produced with a given velocity}}". In this formula, the "velocity" <math>n</math> is what is now more commonly denoted with the Greek letter <math>\omega.</math> &nbsp;--[[User talk:Lambiam#top|Lambiam]] 06:49, 14 May 2024 (UTC)
::::p 540 onwards in Mathematical and Physical Papers, Volume 1 William Thomson Baron Kelvin University Press, 1882 - Mathematics - 619 pages, which is in google books, certainly discusses oscillatory behavior and time between peaks but I don't think it explicitly states f=1/(2pi*sqrt(L*C)). Particularly equation 7 where his A is modern L. [[User:Greglocock|Greglocock]] ([[User talk:Greglocock|talk]]) 00:08, 14 May 2024 (UTC)


= May 14 =
:::Such projects have existed (in both commercial and non-commercial forms) but I think they have not had a lot of success. [[User:Nimur|Nimur]] 00:34, 29 May 2007 (UTC)


== Why is Ostrich meat Red? ==
== Cranial size and [[Pb]] poisoning ==


Hello, can [[lead poisoning]] affect cranial size?[[User:Richard L. Peterson|Rich]] ([[User talk:Richard L. Peterson|talk]]) 05:43, 14 May 2024 (UTC)
If it is part of the bird family, why is Ostrich Meat Red and not the same as a Turkey or chicken, perhaps this is a trivial question but, I would like all the facts on this matter for peac of mind.
:Yes.<sup>[https://pubmed.ncbi.nlm.nih.gov/10023796/] [https://pubmed.ncbi.nlm.nih.gov/18507499/] [https://jamanetwork.com/journals/jama/fullarticle/2772961]</sup> --[[Special:Contributions/136.54.106.120|136.54.106.120]] ([[User talk:136.54.106.120|talk]]) 11:40, 14 May 2024 (UTC)
{{unsigned|41.241.11.179}}


= May 15 =
: See [[White meat]] article. [[User:Dr_Dima|Dr_Dima]].


== They see me rollin', they hatin... ==
::That doesn't really answer the question. Ostrich meat is not dark like chicken drumsticks; it looks like beef. I'd like to know why, also. I would guess it has something to do with the fact that ostriches do not fly. --[[User:Tugbug|Tugbug]] 20:17, 28 May 2007 (UTC)


Question for you guys. Is there any animal whose primary method of locomotion is curling into a ball and rolling head over heels to get around? Rather than running or walking. Because I think some woodlice do it (faster for them to roll then run), but I'm not 100% sure. [[User:Iloveparrots|Iloveparrots]] ([[User talk:Iloveparrots|talk]]) 03:04, 15 May 2024 (UTC)
== Lagrange points with other bodies nearby ==
: Do [[Curl-up|fictional animals]] count? --[[Special:Contributions/136.54.106.120|136.54.106.120]] ([[User talk:136.54.106.120|talk]]) 03:33, 15 May 2024 (UTC)
:[[Terrestrial locomotion #Rolling]] may be of interest. --[[Special:Contributions/136.54.106.120|136.54.106.120]] ([[User talk:136.54.106.120|talk]]) 03:38, 15 May 2024 (UTC)
:I don't see how that could ever be a ''primary'' means of locomotion, but see also [[:Category:Rolling animals]]. [[User:Shantavira|Shantavira]]|[[User talk:Shantavira|<sup>feed me</sup>]] 08:37, 15 May 2024 (UTC)
:[[Hedgehogs]] do it to escape from predators when threatened. [[Special:Contributions/2601:646:8082:BA0:BC05:6EA8:F933:9E6D|2601:646:8082:BA0:BC05:6EA8:F933:9E6D]] ([[User talk:2601:646:8082:BA0:BC05:6EA8:F933:9E6D|talk]]) 10:43, 15 May 2024 (UTC)
::Hedgehogs roll up for protection, not for locomotion. (Ditto for the woodlice mentioned by the OP). [[User:Wardog|Iapetus]] ([[User talk:Wardog|talk]]) 11:57, 15 May 2024 (UTC)
:::I don't think I've ever seen a hedgehog roll outside of a video game. But then again, I've only ever seen hedgehogs in real life about three times. [[User:Iloveparrots|Iloveparrots]] ([[User talk:Iloveparrots|talk]]) 22:58, 15 May 2024 (UTC)
::::Hedgehogs do not purposely roll when in a defensive ball. They "huff", which makes them bounce, forcing their quills into whatever is attacking them. That bounce could cause a roll, but it isn't on purpose. [[Special:Contributions/75.136.148.8|75.136.148.8]] ([[User talk:75.136.148.8|talk]]) 13:40, 16 May 2024 (UTC)
:See [[Rotating locomotion in living systems]]. [[Special:Contributions/2605:B100:34D:46C3:61A4:6B17:A082:3780|2605:B100:34D:46C3:61A4:6B17:A082:3780]] ([[User talk:2605:B100:34D:46C3:61A4:6B17:A082:3780|talk]]) 12:27, 15 May 2024 (UTC)
:I have encountered assertions that [[Giant panda]]s, when sitting in a bamboo thicket on a slope, will sometimes roll a short distance rather than get up and walk, but this would need confirmation from a reliable source, and in any case would not be a ''primary'' means of locomotion. {The poster formerly known as 87.81.230.195} [[Special:Contributions/94.2.67.173|94.2.67.173]] ([[User talk:94.2.67.173|talk]]) 19:48, 15 May 2024 (UTC)
::A giant panda is rolling head first in this video: [https://www.youtube.com/shorts/hWd8RGuMgE0 Panda Discovers Something Interesting.] They roll about a lot too, [https://www.nathab.com/blog/weird-panda-behavior-explained/ for various reasons]. [[User:Modocc|Modocc]] ([[User talk:Modocc|talk]]) 23:30, 15 May 2024 (UTC)


== What's the max depth of the Baltic, Black and Azov brim? ==
[[Image:Lagrange points.jpg|thumb|[[Lagrange point|Lagrange points]]]]
Why don't the [[Lagrange point|Lagrange points]] of, say, Sun–Earth get ruined by the other bodies in the solar system? Is it that the gravitational pull from the other bodies average out since they don't move along with the Sun–Earth system and therefore pull in different directions at different times? —[[User:Bromskloss|Bromskloss]] 13:41, 28 May 2007 (UTC)


The depth where it stops being connected to the World Ocean 50% of the time (connected only by seepage through porous solids like silt not counting as connected). As the brim can erode, shift if the water's removed, be a V-notch in a ridge etc and even if it's dredged wide, straight and flat it might not be legal to lightly touch the silt so this might not round to the same number of feet as the deepest draft ship that's allowed at least 50% of the Metonic cycle, or how much sea level would have to drop to make it a lake 50% of the time. Also what would the depth be if the strait bottom wasn't landscaped? The Turkish Straits are pretty deep by ship standards maybe they aren't landscaped? [[User:Sagittarian Milky Way|Sagittarian Milky Way]] ([[User talk:Sagittarian Milky Way|talk]]) 18:05, 15 May 2024 (UTC)
:As explained at [[Lagrangian point#Stability]], the effect of the rest of the solar system on Lagrange points is enough to complicate the orbits quite a bit, but (the rest of the solar system being quite light and a long way away) is not enough to de-stabilize the L<sub>4</sub> and L<sub>5</sub> points. It ''does'' destablize the orbits about the other three points, but this is small enough to just require slight correcting maneouvres every so often. [[User:Algebraist|Algebraist]] 17:05, 28 May 2007 (UTC)


:They wouldn't need to be "landscaped", the [[Bosphorous]] is typically about 60 m deep with a 40 m sill towards its southern end, although the Asian side of the strait at that point is somewhat shallower (about 27 m) - see [https://agupubs.onlinelibrary.wiley.com/doi/full/10.1029/2003PA000903 Siddall et al. (2004)]. [[User:Mikenorton|Mikenorton]] ([[User talk:Mikenorton|talk]]) 19:11, 15 May 2024 (UTC)
:: OK, but the other objects aren't necessarily lighter and farther away than Earth, right? —[[User:Bromskloss|Bromskloss]] 19:45, 28 May 2007 (UTC)
::Right I didn't think they would. Unless the depth of the rim's now artificial by more than a foot due to sinking accidents(s), full or partial intentional blockages or explosion(s) (possibly to clear some of the previous)? Or maybe it's still the natural rock or sediment accumulation-erosion surface? I don't know if the Baltic and Azov brim are unaffected by human landscaping. [[User:Sagittarian Milky Way|Sagittarian Milky Way]] ([[User talk:Sagittarian Milky Way|talk]]) 20:16, 15 May 2024 (UTC)
:::Well, they are all much further away - they may not be lighter though - the outer planets are all pretty huge. Since the gravitational forces due to those more distant bodies is inversely proportional to the SQUARE of the distance - their effect diminishes rapidly as they get further away. The distinctive factor about L4 and L5 (as opposed to the other Lagrange points) is that they form peculiar (unless you are a mathematician!) 'virtual' gravity sources - if you are close to one of those empty points in space, you'll get pulled towards them - you can even orbit them! This makes an object at one of those two points fairly insensitive to small disturbances - since this virtual gravity well will pull them back in again. But at the L1/2/3/6 points, the balance between the various forces is unstable - if you are exactly at the Lagrange point, you're theoretically in perfect balance between gravity of Earth and Moon and the "centrifugal" forces involved - but any small displacement from that point will result in you being pulled further and further away. So at (for example) L1, a teeny tiny occasional tug from (say) Jupiter - however small - will be enough to gradually move you away - and eventually you'll smack into the Moon or the Earth or get flung out of the Earth/Moon system altogether. [[User:SteveBaker|SteveBaker]] 20:00, 28 May 2007 (UTC)


:::: You say others are further away, but what exactly do you mean? There are surely instants when other planets are closer to one of the points than both Sun and Earth. —[[User:Bromskloss|Bromskloss]] 20:26, 28 May 2007 (UTC)


= May 17 =
:::::Actually - I didn't read the question properly. I was thinking about the earth/moon lagrange points - not the Earth/sun. But still, the L1/2/3/6 points are unstable and the L4/5 points are stable - so in all likelyhood the same thing applies. [[User:SteveBaker|SteveBaker]] 22:16, 28 May 2007 (UTC)


== What would cause these 'dark area' blemishes on an LCD monitor? ==
== Radioactivity ==


See this photo: https://ibb.co/mz8vQh0
Knowing the half-life of a substance like Polonium-210 or Plutonium-239, how do I calculate how many alpha particles would be ejected, theoretically on average, per hour or so from a given sample size (say, 1 microgram of each)? --[[User:140.247.240.18|140.247.240.18]] 14:17, 28 May 2007 (UTC)


This is my Asus Designo MX25AQ main monitor, I've had it for a little over four years and two months now. Since about a year ago, it started developing this issue where a "wavy" area of darkness appears near the left and right edges of the screen. It looks as if there's liquid in the screen or if something's "delaminating" inside. It would usually happen when it's cold, and would go away / "fade away" as it warmed up (usually taking a few minutes). However, over the last few months, it's been getting worse and worse. Nowadays, sometimes it's visible on certain shades of colours even when the monitor is fully warmed up. It is especially noticeable when it cools down in real life (e.g. it's becoming dawn and the brightness has been decreased).
:You start by computing the number of atoms in a microgram using the mass of a single polonium atom. Then you convert the half life to a decay rate by using the formula you can find in [[Radioactive_decay]] under the section decay timing. Now compute the number of atoms that are left after one hour using the exponential decay formula slightly above in the same article. The difference between both numbers is the number of atoms that decayed. The number of aplha particles is the same, provided that the decay sequence does not produce multiple alphas per atom.


The "grey uniformity" of the monitor has been degrading a bit as well, I swear.
::That sounds reasonable. Thanks! --[[User:140.247.240.137|140.247.240.137]] 19:26, 28 May 2007 (UTC)


I know I likely won't be able to fix an issue like this with the LCD panel. But my question is, what would possibly cause these issues? Could it develop into a stage where my LCD becomes completely / severely broken?
== Distillation of ethanol, temperature ==


Note that I'm posting this to RD/S and not RD/C because I strongly believe this issue has something to do with materials degradation in the LCD and not some issue with the computer or cable, so I figured this is a better place for that.
When a mixture of ethanol and water is heated and reaches the boiling point of ethanol, will the temperature stay the same (approx. 78.5°C) until all the ethanol has evaporated, or is it possible to heat such a mixture to, say 85°C, at ambient pressure? --[[User:62.16.173.45|62.16.173.45]] 14:34, 28 May 2007 (UTC)


Apologies for the non-free external image site upload, but if everything you see in the photo is not copyrighted then I will upload it to Commons. —&nbsp;[[User:AP 499D25|<span style="background:#1F6295;color:white;padding:1q 5q;border-radius:10q;font-family:Franklin Gothic, Verdana">AP&nbsp;499D25</span>]] [[User talk:AP 499D25|<span style="color:#1A527D">(talk)</span>]] 05:03, 17 May 2024 (UTC)
:The mixture will approximately stay at the same temperature until the ethanol has evaporated. It is not totally accurate, there are some effects when you go into detail. [[Destillation]] talks about the variation of the boiling point of ethanol in a solution. There is also the possibility of [[Superheating]].
:I Googled around, and it could be moisture infiltrating around the edges. The fact that it clears when hot is suggestive of that. There was a suggestion that if the monitor is near a kitchen, these stains could include cooking fumes. Another possibility (and conceivably related) is damage from pulling the protective plastic sheet off, the one that came when the monitor was newly purchased. This has to be done extremely carefully. Yet another worry is cleaning with rubbing alcohol, the internet says this is a bad idea. <span style="font-family: Cambria;"> [[User:Abductive|<span style="color: teal;">'''Abductive'''</span>]] ([[User talk:Abductive|reasoning]])</span> 06:11, 17 May 2024 (UTC)
:::{{small|[[Rubbing alcohol]] as sold can consist for up to 50% of water. &nbsp;--[[User talk:Lambiam#top|Lambiam]] 09:16, 18 May 2024 (UTC)}}
::That sounds plausible! Thinking about it, more than a year ago I did clean my monitor using a wet microfibre cloth (a small, thin one for phones), and that seems to be the timepoint it all started. Worth pointing out that this monitor is in a bedroom, so it's not near moisture or oil vapour sources, and it has never been used in such an environment. When I got it new, there wasn't actually a protective sticky film on the front, just a styrofoam wrap. When I slid that wrap off, I got a nice big static shock when I touched the monitor bezel, but that didn't seem to do any immediate damage – this monitor was fine for the first 2.5 years or so that I used it.
::Another interesting fact about this monitor is that although I bought it in Mar 2020, according to the info label, it has an manufacture date of Sep 2015, so that means it sat in a warehouse for 4.5 years before I bought it I guess.
::Aside from this annoying and distracting issue, this monitor is the best quality display I've ever had (QHD resolution and 100% sRGB accuracy), so it'd be such a shame if it's actually dying on me. —&nbsp;[[User:AP 499D25|<span style="background:#1F6295;color:white;padding:1q 5q;border-radius:10q;font-family:Franklin Gothic, Verdana">AP&nbsp;499D25</span>]] [[User talk:AP 499D25|<span style="color:#1A527D">(talk)</span>]] 08:06, 17 May 2024 (UTC)


== Is there any scientific truth to the meme that Vegetable oil/Seed oil is toxic to the human body? ==
::Thank you! --[[User:62.16.173.45|62.16.173.45]] 18:15, 28 May 2007 (UTC)
There are plenty of memes that seed oil causes diabetes and heart attacks but I failed to find any wikipedia articles that argues their case. Is there any scientific truth to the meme that Vegetable oil/Seed oil is toxic/harmful to the human body?


Another question is that if the meme of harmful seed oil is unscientific then why isn't this meme documented in the List_of_conspiracy_theories wiki page? [[Special:Contributions/2001:8003:429D:4100:A593:8A5B:182E:5551|2001:8003:429D:4100:A593:8A5B:182E:5551]] ([[User talk:2001:8003:429D:4100:A593:8A5B:182E:5551|talk]]) 15:55, 17 May 2024 (UTC)
== ARUN ARUN QUES ABOUT PENIS ==
:Any kind of fats or oils '''in excess''' can cause heart disease, but there is '''no''' truth to the claim that vegetable oil is more toxic than animal fats (in fact, it's actually healthier) or that its consumption '''in moderation''' causes any health problems. [[Special:Contributions/2601:646:8082:BA0:9480:50AE:ABF3:5E17|2601:646:8082:BA0:9480:50AE:ABF3:5E17]] ([[User talk:2601:646:8082:BA0:9480:50AE:ABF3:5E17|talk]]) 23:30, 17 May 2024 (UTC)


:Yes, there is truth to it. Our nutrition articles could do with some updating and balance. It is a prime example of Paracelsus's [[The dose makes the poison]], for both the omega 6 LA and omega 3 ALA are essential nutrients for humans. The basic issue is consumption of a high quantity of [[omega-6 fatty acid]]s from modern seed oils (which have only been consumed for a century or so) and other sources, including indirectly through animal feed and the (relative) paucity of [[omega-3 fatty acid]]s in the modern human diet. [Excessive omega-3 over omega-6 has been found only in [[Greenlandic Inuit]] traditional communities.] Probably the best book to start with is Anthony John Hulbert's recent Omega Balance: Nutritional Power for a Happier, Healthier Life- Johns Hopkins (2022). By omega balance, he means the percentage of omega 3's in the sum of omega 3's and 6's. He says:
to what extent I should retract my foreskin to wash smegma?


<blockquote>Although there is no advice about separate consumption of omega-3 and omega-6 fats in these national dietary guidelines, this is not the case with the premier scientific society concerning lipid research. In 2004, ISSFAL ([[International Society for the Study of Fatty Acids and Lipids]]) issued a series of recommendations for dietary intake of the essential fats by healthy adults. They made no comment about consumption of the nonessential saturated and monounsaturated fats but instead proposed that adequate intake of 18:2ω-6 [ [[Linoleic acid]] (LA) ] is 2 percent of energy, and a healthy intake of 18:3ω-3 [ [[α-linolenic acid|α-linolenic acid (ALA)]] ] is 0.7 percent of energy as well as recommending a minimum intake of 500mg/d of 20:5ω-3 [ [[eicosapentaenoic acid]] (EPA)] and 22:6ω-6 [sic, should be 22:6ω-3 [[docosahexaenoic acid]] (DHA) as in the source [https://www.issfal.org/assets/issfal%2003%20pufaintakereccomdfinalreport.pdf]]. These ISSFAL recommendations for daily intake correspond to a diet omega balance of about 30 percent.
:You should never retract your foreskin in a way that feels in painful to you. If you are still growing up, your foreskin may not be fully retractable. That is completely normal.
The recommended intakes contrast markedly with the average actual daily intakes by the US population (from a 1999–2000 survey), which correspond to a diet omega balance of 9 percent. Similarly, a dietary survey of the Australian population revealed the average daily intake in 1995 corresponded to a diet omega balance of 11 percent. Both the United States and Australia (and likely many other developed high-income countries) have omega-6 intakes much higher and omega-3 intakes lower than the recommended levels.</blockquote>


:Hulbert and other sources provide evidence that the omega imbalance can have deleterious effects not only in various chronic diseases, but also that the excess of inflammatory omega-6's can worsen outcomes of Covid, where many deaths appear to come from an excessive inflammatory response.[[User:John Z|John Z]] ([[User talk:John Z|talk]])
:Right. The condition of the foreskin not being fully rectractable is called [[phimosis]], and our article on the subject states that it is common for boys and vanishes when growing up. For older teenagers and adults, it can, however, cause inconvenience or pain. So if you are still young and it is not painful, don't worry, otherwise, better go and ask a doctor for advice. And don't get worried by the [[phimosis]] article -- to my taste it is a bit technical and medical, so if you really want to get clear information, don't feel embarrassed to simply ask a doctor. [[User:Sanders muc|Simon A.]] 17:09, 28 May 2007 (UTC)


:[[Water|μ-Oxidodihydrogen]], a chemical compound found in industrially processed [[canned soup]], is also known to be [[Water intoxication|toxic to the human body]]. Why is no one talking about this? &nbsp;--[[User talk:Lambiam#top|Lambiam]] 09:10, 18 May 2024 (UTC)
::Our article on phimosis is really a bit strange. I would suggest to read this instead: http://www.nocirc.org/publish/pamphlet4.html.


:In very large excess, such as Olympic sized swimming pools, you would find it extremely difficult to swim in oil. [[User:Graeme Bartlett|Graeme Bartlett]] ([[User talk:Graeme Bartlett|talk]]) 12:37, 20 May 2024 (UTC)
== Red Rice - Health benefits ==
::{{small|It might be even more difficult in a bathtub. &nbsp;--[[User talk:Lambiam#top|Lambiam]] 19:31, 20 May 2024 (UTC)}}
:::A wry comparison to the dangers of water is appropriate to much or most discussion of nutrition. Which proceeds by demonizing food X and then along with declaring it causes disease Y, X is declared to be a novelty even though it was consumed by all or much of humanity for tens of millennia at least, or is even essential to life. Historical absurdity is absurdly accepted. But it isn't relevant here because nobody considers modern seed oils to be anything but essentially new foods, never existing or consumed in such bulk before by humans or by any animal. And there aren't many such candidates for widespread dietary changes that could be implicated in global rises of chronic diseases.
:::Another source, which may conceal some such wryness, is [https://archive.org/details/preventionofcoro0096unse Harumi Okuyama, Yuko Ichikawa, Yueji Sun, Tomohito Hamazaki, William Edward Mitchell Lands- Prevention of Coronary Heart Disease: From the Cholesterol Hypothesis to omega 6 omega 3 Balance- Karger (2006)]. They note that "Dietary advice was revealed to be the most serious risk factor for CHD in Japan." "We suggest that increased intake of LA [a consequence of that advice as it raised seed oil consumption] may be a major cause for the observed increase in CHD incidence in the group with dietary advice. Higher intakes of LA accompanied higher rates of CHD ( fig. 9–11 ; tables 4 , 5 ), whereas decreasing LA intake was effective for the secondary prevention of CHD events". In any case, there is a genuine, active scientific controversy here, not a conspiracy theory. Those who see omega imbalance as a real problem - and therefore seed oils, which uncontroversially are its ultimate source- may be the majority of specialist lipidologists, e.g. [[Artemis Simopoulos]], cofounder of ISSFAL.[[User:John Z|John Z]] ([[User talk:John Z|talk]]) 04:20, 21 May 2024 (UTC)


== Butterfly size ==
What are the health benefits of the Red Rice available in India? Is the nutrition value the same as Brown Rice? - Priya


Do butterflies (especially nymphalids and/or swallowtails) become significantly smaller in size near the poleward (high-latitude) limit of their natural range? When I visited the [[Royal Ontario Museum]] in [[Toronto]], they had 2 pinned specimens of [[Papilio cresphontes]] on display which were '''much''' smaller than their normal size per the article (one had a wingspan of "only" 3 inches -- I did a rough measurement with my fingers against the glass -- and the other was about 1/2 inch bigger) -- is this normal for (1) specifically ''P. cresphontes'', (2) all swallowtails, and/or (3) all or most butterfly species? [[Special:Contributions/2601:646:8082:BA0:9480:50AE:ABF3:5E17|2601:646:8082:BA0:9480:50AE:ABF3:5E17]] ([[User talk:2601:646:8082:BA0:9480:50AE:ABF3:5E17|talk]]) 23:38, 17 May 2024 (UTC)
:The [[red rice]] article claims that it is a "pest" or weed, because less edible grains grow per plant. I don't know if it is actually unhealthy. [[User:Nimur|Nimur]] 00:38, 29 May 2007 (UTC)


:[https://www.jstor.org/stable/3546281 ''Size Distributions of Butterfly Species and the Effect of Latitude on Species Sizes''] (you can open a free JSTOR account or access through the [https://wikipedialibrary.wmflabs.org/?next_url=/users/my_library/ Wikipedia Library]). [[User:Alansplodge|Alansplodge]] ([[User talk:Alansplodge|talk]]) 10:10, 18 May 2024 (UTC)
== Regression analysis for tidal water levels ==
::Sorry, I won't click on any external links regarding '''this''' topic, just in case it might show me gratuitously enlarged pictures of ''P. multicaudata'' or some suchlike abomination -- can you just tell me the gist of it in a few words (or more than a few, your choice)? [[Special:Contributions/2601:646:8082:BA0:E558:16C8:D2DE:51EF|2601:646:8082:BA0:E558:16C8:D2DE:51EF]] ([[User talk:2601:646:8082:BA0:E558:16C8:D2DE:51EF|talk]]) 10:31, 18 May 2024 (UTC)
:::There are no pictures, it's a scientific paper. "For butterfly species (Papilionoidea) of the Australian and Afrotropical regions, average wingspan decreases with increasing latitude". [[User:Alansplodge|Alansplodge]] ([[User talk:Alansplodge|talk]]) 10:34, 18 May 2024 (UTC)
::::Thanks! So the answer is yes -- right? (And that would explain the unusually small size of the two ''P. cresphontes'' specimens at the museum -- they must have been caught locally, and Toronto is near the northern limit of this species' natural range! And that is also quite reassuring for me -- it means that in [[Portland, Maine]] where I've been planning to move for quite a while, any ''P. cresphontes'' I come across won't be scary huge, in fact I might actually come to like them!) [[Special:Contributions/2601:646:8082:BA0:E55E:2854:FEDE:FEB6|2601:646:8082:BA0:E55E:2854:FEDE:FEB6]] ([[User talk:2601:646:8082:BA0:E55E:2854:FEDE:FEB6|talk]]) 21:21, 18 May 2024 (UTC)


= May 18 =
I'm doing some studying on the behavior of tides. I've made a big table of water levels at different times over two days, and I made a big graph of it. Of course, the tides go according to a sinusoidal function, but both the gravitational effects of the Moon and the Sun affect tidal water levels, so therefore, the function of water levels would be in this form:


== Why packaging is important ==
(where L is water Level and t is Time)


Two reasons why packaging is important [[User:PhuPhumzile|PhuPhumzile]] ([[User talk:PhuPhumzile|talk]]) 07:10, 18 May 2024 (UTC)
L = A + B sin(C(t)) + D (sin E(t + F))


:
Where A would be the mean water level, B would be the amplitude of the lunar component of water levels, C would represent the period of the moon, D would be the amplitude of the solar component to water levels, E would represent the period of the sun (it would be equal to 2π*24), and F would be an offset between the start of the lunar cycle and start of the solar cycle.
:[[File:Pictogram voting delete.svg|20px]] '''Please [[WP:DYOH|do your own homework]].'''
:Welcome to {{#ifeq:{{BASEPAGENAME}}|Help desk|[[Wikipedia:Help desk|the Wikipedia Help Desk]]|{{#ifeq:{{BASEPAGENAME}}|Reference desk|[[Wikipedia:Reference desk|the Wikipedia Reference Desk]]|Wikipedia}}}}. Your question appears to be a homework question. I apologize if this is a misinterpretation, but it is [[Wikipedia:Do your own homework|our aim here]] not to do people's homework for them, but to merely aid them in doing it themselves. Letting someone else do your homework does not help you learn nearly as much as doing it yourself. Please attempt to solve the problem or answer the question yourself first. If you need help with a specific part of your homework, feel free to tell us where you are stuck and ask for help. If you need help grasping the concept of a problem, by all means let us know.<!--Template:Dyoh--> [[User:Shantavira|Shantavira]]|[[User talk:Shantavira|<sup>feed me</sup>]] 09:16, 18 May 2024 (UTC)
*Read our article on "[[Packaging]]". It may assist you. [[User:Graeme Bartlett|Graeme Bartlett]] ([[User talk:Graeme Bartlett|talk]]) 09:40, 18 May 2024 (UTC)
::Ya gotta love it when a poster doesn't even try to make it look like a question they thought of on their own. Or like a question, even. ←[[User:Baseball Bugs|Baseball Bugs]] <sup>''[[User talk:Baseball Bugs|What's up, Doc?]]''</sup> [[Special:Contributions/Baseball_Bugs|carrots]]→ 14:23, 18 May 2024 (UTC)
:::{{small|One reason: By packaging your homework question to make it look as if it is curiosity-driven, you have a much better chance of getting a useful answer. &nbsp;--[[User talk:Lambiam#top|Lambiam]] 14:41, 18 May 2024 (UTC)}}
::Why packaging is important: try buying loose helium by weight. Or by the handful. [[User:AndyTheGrump|AndyTheGrump]] ([[User talk:AndyTheGrump|talk]]) 15:30, 18 May 2024 (UTC)
:In certain forms [[shrinkflation]], despite selling less of a product, manufacturers keep the packaging at the original size and leave some of the space empty. This wrapping and transportation of air maintains the level of economic activity, which is important to the [[gross domestic product]]. [[User:Card_Zero|<span style=" background-color:#fffff0; border:1px #995; border-style:dotted solid solid dotted;">&nbsp;Card&nbsp;Zero&nbsp;</span>]]&nbsp;[[User_talk:Card_Zero|(talk)]] 21:22, 20 May 2024 (UTC)


= May 19 =
I'm trying to figure out the numerical values of all those lettered coefficients. Now, I could figure all of those out with astronomical/oceanographic data (the period of the sun, the moon, the mean water level) besides B and D. For those two, I'd need to do a regression analysis from the data I tabulated earlier.


== Erythema Migrans ==
I don't know how to do regression analysis by hand (though I guess I could figure it out or try to learn it), so I looked all over the internet for a regression analysis program. I found a bunch that do regression analysis, but none of them do sinusoidal regression analysis. A polynomial regression analysis would work fairly well except that to find out the corresponding sinusoidal function, I'd have to do some super-complicated reverse Taylor series...


Hi
So my question is: Does anyone know a freeware/shareware/demo program that can do a sinusoidal regression analysis for something like this? If not, is there an easy way (without spending hours messing with taylor series) to figure out how to find the corresponding sinusoidal-sum equation from a polynomial equation? [[User:Jolb|Jolb]] 16:53, 28 May 2007 (UTC)


Does anyone have a good source for erythema migrans? I was looking for another one and can't copy (for my own notes) the Wikipedia page
: First, you won't see the dependency on the Sun with only two days of data. Second, local conditions such as the shape of the coastline may strongly affect the data. Now to your question: Whenever you have function which you thin is composed of a sum of sines with different amplitudes and phases, you want to do a [[Fourier analysis]]. I am sure that there is free software to do this, but you can also do it easily with even only a little programming skills, or even with a [[spreadsheet]] program. Unfortunatly, our articles on [[Fourier transform]] and [[Fourier analysis]] do not breally strike me as very accessible for beginners, so a good textbook on engineering math might be a better choice to learn about it. But for a starting point, try the following (and then learn from your textbook, why this does what it does): Assuming that you have measured the tide at ''N'' times ''t<sub>i</sub>'' and got the values ''h<sub>i</sub>'', calculate


[[User:W&#59;ChangingUsername|W&#59;ChangingUsername]] ([[User talk:W&#59;ChangingUsername|talk]]) 19:04, 19 May 2024 (UTC)
: <math>W_c(T) = \frac{1}{N} \sum_{i=1}^N h_i \cos (2\pi t_i/T) </math>
:[[Erythema migrans]]. ←[[User:Baseball Bugs|Baseball Bugs]] <sup>''[[User talk:Baseball Bugs|What's up, Doc?]]''</sup> [[Special:Contributions/Baseball_Bugs|carrots]]→ 20:19, 19 May 2024 (UTC)
::I just came from there. The article isnt very good im sorry [[User:W&#59;ChangingUsername|W&#59;ChangingUsername]] ([[User talk:W&#59;ChangingUsername|talk]]) 21:02, 19 May 2024 (UTC)
:::[https://scholar.google.com/scholar?hl=en&as_sdt=0%2C14&q=%22Erythema+Migrans%22+%22review%22&btnG= This Google Scholar search] reduces the number of results from 19,400 to 11,900 by adding the term "review". <span style="font-family: Cambria;"> [[User:Abductive|<span style="color: teal;">'''Abductive'''</span>]] ([[User talk:Abductive|reasoning]])</span> 11:10, 20 May 2024 (UTC)
::::Thank you :)
::::I'll probably start using this for everything [[User:W&#59;ChangingUsername|W&#59;ChangingUsername]] ([[User talk:W&#59;ChangingUsername|talk]]) 18:50, 20 May 2024 (UTC)


= May 20 =
: and


== Can testosterone boost/etc... change someone desire from responsive to spontaneous? ==
: <math>W_s(T) = \frac{1}{N} \sum_{i=1}^N h_i\sin (2\pi t_i/T),</math>


Can testosterone boost/etc... change someone desire from responsive to spontaneous?[[Special:Contributions/177.207.104.19|177.207.104.19]] ([[User talk:177.207.104.19|talk]]) 01:37, 20 May 2024 (UTC)
: where ''T'' is time length that you expect to be a period. Change ''T'' a bit, from a bit less than half a day to a bit more than it. When is <math>W_c^2+W_s^2</math> maximal? If you have more data, try also with values for ''T'' around one month. Have fun. [[User:Sanders muc|Simon A.]] 17:22, 28 May 2007 (UTC)
:What do you mean? ←[[User:Baseball Bugs|Baseball Bugs]] <sup>''[[User talk:Baseball Bugs|What's up, Doc?]]''</sup> [[Special:Contributions/Baseball_Bugs|carrots]]→ 01:39, 20 May 2024 (UTC)
:::<small>See [[Sexual desire]] and various other related articles, which you could have found easily by putting 'responsive desire' into the search box of this encyclopedia. You often post similar nonproductive responses to things which you personally have not heard of, although many others have: it becomes tedious. {The poster formerly known as 87.81.230.195} [[Special:Contributions/94.2.67.173|94.2.67.173]] ([[User talk:94.2.67.173|talk]]) 11:58, 20 May 2024 (UTC)
::::I see no harm in trying to encourage posters to link to what they're asking about. ←[[User:Baseball Bugs|Baseball Bugs]] <sup>''[[User talk:Baseball Bugs|What's up, Doc?]]''</sup> [[Special:Contributions/Baseball_Bugs|carrots]]→ 12:44, 20 May 2024 (UTC)
:::::To be fair, there was a non-zero chance that it referred to a responsive to spontaneous change in the desire to fight strangers. [[User:Sean.hoyland|Sean.hoyland]] ([[User talk:Sean.hoyland|talk]]) 13:01, 20 May 2024 (UTC)
::::::Theoretically. The funny thing is that the first sentence of 94.2.67.173's lecture to me could just as easily have served as a direct response to the poster. (Though maybe that was the point anyway!) ←[[User:Baseball Bugs|Baseball Bugs]] <sup>''[[User talk:Baseball Bugs|What's up, Doc?]]''</sup> [[Special:Contributions/Baseball_Bugs|carrots]]→ 14:40, 20 May 2024 (UTC) </small>


::You may be thinking of the [[Dopamine receptor D5|DRD5]] gene, where some alleles are thought to be involved in spontaneity or lack thereof. <span style="font-family: Cambria;"> [[User:Abductive|<span style="color: teal;">'''Abductive'''</span>]] ([[User talk:Abductive|reasoning]])</span> 11:15, 20 May 2024 (UTC)
== Blind sugar-eating dogs ==
:::Our article doesn't mention that, but implicates it in everything else: learning and memory, addiction, smoking, ADHD, Parkinson's disease, schizophrenia, locomotion, regulation of blood pressure, and immunity. What a busy gene. I guess you're referencing something along the lines of dopamine being important for the will to initiate movement, like in [[Awakenings]]. [[User:Card_Zero|<span style=" background-color:#fffff0; border:1px #995; border-style:dotted solid solid dotted;">&nbsp;Card&nbsp;Zero&nbsp;</span>]]&nbsp;[[User_talk:Card_Zero|(talk)]] 21:32, 20 May 2024 (UTC)
:According to [https://link.springer.com/article/10.1007/s10508-012-9946-2 this review article], "{{tq|of the few studies on T}}[estosterone] {{tq|and desire in healthy women ... dyadic desire}} [desire in sex with a partner] {{tq|has shown null or negative correlations with T}}". This suggests it is unlikely to achieve the specific effect. &nbsp;--[[User talk:Lambiam#top|Lambiam]] 08:58, 21 May 2024 (UTC)


== [[Gregory Cochran]] ==
A friend of mine once told me that sugar makes dogs go blind. Is that true? --[[User:Taraborn|Taraborn]] 18:23, 28 May 2007 (UTC)
:It seems really unlikely - dogs are going to get sugars in their diet no matter how healthily you feed them. I guess the most likely issue is if the dog is diabetic. Blindness is a commom symptom of untreated diabetes - and it's likely to be exacerbated by increased dietary sugar. But a healthy (or at least non-diabetic) dog shouldn't have any problems. None of the dog books I have mention sugar in their long lists of 'normal foods' that can be lethal to dogs (chocolate and grapes for example). [[User:SteveBaker|SteveBaker]] 19:49, 28 May 2007 (UTC)


Has Gregory M. Cochran worked for [[Darpa]]? If so, in what capacity, if known?[[User:Richard L. Peterson|Rich]] ([[User talk:Richard L. Peterson|talk]]) 21:45, 20 May 2024 (UTC)
::Is chocolate toxic to gulls, as a matter of interest? I've always erred on the side of caution and never given them any food that's remotely chocolatey. I know that it's toxic to parrots. Anyone know? --[[User:Kurt Shaped Box|Kurt Shaped Box]] 21:04, 28 May 2007 (UTC)
:[https://www.edge.org/memberbio/gregory_cochran This Gregory Cochran] would be a likely candidate, but I could find no direct evidence to support this. This [[Gregory Cochran|Gregory M. Cochran]] seems less likely. There was a '[https://webarchive.library.unt.edu/eot2008/20090114015859/http:/www.darpa.gov/coinlog.html Doug Cochran]' at DARPA, however. --[[Special:Contributions/136.54.106.120|136.54.106.120]] ([[User talk:136.54.106.120|talk]]) 01:09, 21 May 2024 (UTC)
::I bet those 2 Gregory Cochrans are the same.[[User:Richard L. Peterson|Rich]] ([[User talk:Richard L. Peterson|talk]]) 02:27, 21 May 2024 (UTC)
::: You're correct: they each are "co-author of the book ''The 10,000 Year Explosion''." That ''Edge'' link could be used as a source for updating the article. --[[Special:Contributions/136.54.106.120|136.54.106.120]] ([[User talk:136.54.106.120|talk]]) 05:57, 21 May 2024 (UTC)


= May 21 =
:::Yes, [[theobromine]], the stimulant component of chocolate (often mistaken for [[caffeine]], which is only present in chocolate in trace ammounts), causes [[theobromine poisoning]] in many animals (dogs, horses, parrots) because they are unable to metabolise it quick enough. [[User:Smurrayinchester|<span style="color:#BB0055">Laïka</span>]] 22:05, 28 May 2007 (UTC)


== Butterflies of Daviess County, KY (Papilionidae) ==
::::I wonder what the toxic dose for a human would be? --[[User:Kurt Shaped Box|Kurt Shaped Box]] 22:21, 28 May 2007 (UTC)
:::::See [[Death by Chocolate]]? ;) --[[User:Krsont|Krsont]] 22:40, 28 May 2007 (UTC)


In [[Daviess County]], [[Kentucky]] (or generally along the [[Ohio River]] or within a reasonable distance south thereof), how late in the year was the latest-ever sighting of ''Papilio glaucus''? (Here in Central California, all tiger swallowtail species disappear in the first days of September -- my latest confirmed sighting of ''P. multicaudata'' was on September 1, and of ''P. rutulus'' on September 4 -- is it more-or-less the same over there?) Asking for a local and/or an expert -- and '''no pictures, please'''! [[Special:Contributions/2601:646:8082:BA0:250E:98C8:7461:C819|2601:646:8082:BA0:250E:98C8:7461:C819]] ([[User talk:2601:646:8082:BA0:250E:98C8:7461:C819|talk]]) 02:48, 21 May 2024 (UTC)
== Is it possible to get to proxima centauri in a practicle timescale? ==


:The pool of active RefDesk editors is quite small and is spread around the Anglosphere (or sometimes outside of it), so finding anyone from that locality or an entomologist here is a bit unlikely. A Google search only found [https://www.backyardecology.net/eastern-tiger-swallowtail-butterflies/ backyardecology.net - ''Eastern Tiger Swallowtail Butterflies''] (with pictures) which says: "In Kentucky, we typically see the adults flying from April until September". [[User:Alansplodge|Alansplodge]] ([[User talk:Alansplodge|talk]]) 14:33, 21 May 2024 (UTC)
I did some research about faster than light travel. It made me think that interstellar travel was possible, though it is overwelmingly expinsive. Is most of the second sentence variable? '''[[User:Fbs. 13|Fquantum]]''' <sup>[[User Talk:Fbs. 13|talk]]</sup> 20:48, 28 May 2007 (UTC)
:You don't need to travel faster than light to travel to nearby stars - it just takes a while. But yes, it'd still be prohibitively expensive, since you'd need a large ship for a voyage that takes a generation (at least). [[User:WilyD|WilyD]] 20:51, 28 May 2007 (UTC)
:: Even at the speed of light it would take 4 years (though it would seem less to those actually travelling, per [[relativity]]). It's way beyond our current technology to get there. With our fastest spaceship it will take 17,000 years. See [[interstellar travel]]. --[[user:h2g2bob|h2g2bob]] ([[user talk:h2g2bob|talk]]) 21:18, 28 May 2007 (UTC)
::: [[Interstellar Travel]] is not a good place to look for sources, see one of the categories for details. [[Spacecraft propulsion]] would be a better place to look for sources. Thanks anyway.
:: Our fastest ship would arrive there a useless hunk of junk since the reliability of our engineering isn't good enough to last 17,000 years without maintenance. The solar sail approach (with massive lasers doing the work from orbit somewhere) is probably the way to go - but we don't have the technology or the funding or the will or enough knowledge of the destination to do that. Right now, we're better off building HUGE orbital telescopes - or perhaps a telescope array on the back side of the moon - we'll get more information about more stars more quickly and more cheaply than with robotic probes. [[User:SteveBaker|SteveBaker]] 22:12, 28 May 2007 (UTC)


::Thanks, but I was hoping for a bit more detail -- do they fly until early September (like their close relatives here in California), or until the end of September, or what? (The reason why I ask is, a friend of mine is over there on a farm caring for a sick relative, and I want to come over and [[Farmhand|help her with the farm itself]], but at the same time I want to avoid any chance of a close encounter with one of those creepy critters out in the open (I think I told you more than once before how I feel about the Eastern tiger and about any other butterfly that looks like it -- with the exception of ''P. canadensis'' and ''P. machaon'' because they're nice and small, and also ''P. zelicaon'' because it's not only small but also its stripes are barely visible)! But I think since they only hatch in April (here in California I see ''P. rutulus'' beginning in mid-March), they shouldn't be flying any later in the year than they do here, so I should be "safe" beginning with the second week of September! (And just to be clear, other swallowtails like [[Papilio troilus]] are perfectly fine by me -- only tiger-striped ones aren't!) [[Special:Contributions/2601:646:8082:BA0:A400:D9D:C5FD:AB24|2601:646:8082:BA0:A400:D9D:C5FD:AB24]] ([[User talk:2601:646:8082:BA0:A400:D9D:C5FD:AB24|talk]]) 03:44, 24 May 2024 (UTC)
== Glasses effect ==


:Broadening the topic somewhat, you might be interested (if you aren't already familiar with it) by the subject of [[Phenology]]. Climate change is obviously having a large influence on previously reliable annual timings of natural phenomena: in my part of the world (southern England), many trees are blooming, etc., up to a month earlier than a few decades ago. Doubtless the emergence, migrations and numbers of annual broods of insects are also changing, so they can appear later as well as earlier. {The poster formerly known as 87.81.230.195} [[Special:Contributions/94.2.67.173|94.2.67.173]] ([[User talk:94.2.67.173|talk]]) 09:57, 22 May 2024 (UTC)
Is there any corrective effect of glasses directly on eyes? For instance if one wore glasses for 1 hour, would there be a slight sight improvement without them? --[[User:Brandmeister|Brand]] [[User talk:Brandmeister|спойт]] 21:52, 28 May 2007 (UTC)
::Try calling 270-684-0211, the main number for the county public library in Owensboro. (Lest you embarrass yourself — be aware that the county name is pronounced "Davis", not "Davey's" or "Davy-ess".) Unfortunately their website's "contact" page, https://www.dcplibrary.org/contact, doesn't give either an email address or a form to write a help request. [[User:Nyttend|Nyttend]] ([[User talk:Nyttend|talk]]) 02:18, 23 May 2024 (UTC)
:::Thanks, but I just had a better idea -- is there a zoo and/or a university in Owensboro? Because if so, I think I'll try asking them (and if not, then I'll try Louisville -- the climate is more-or-less the same there, so these critters should disappear at about the same time in both places!) [[Special:Contributions/2601:646:8082:BA0:C564:9993:1EA4:838E|2601:646:8082:BA0:C564:9993:1EA4:838E]] ([[User talk:2601:646:8082:BA0:C564:9993:1EA4:838E|talk]]) 23:17, 24 May 2024 (UTC)


== Pseudouridine - why is it the fifth nucleotide? ==
:I believe that some lenses can train the eye to focus more clearly, although it will depend on why you need glasses and how they are prescribed. I personally wore glasses until around when I was 9, and my vision is actually better than 20/20 now. --<small> [[User:Feba|Phoeba Wright]]<sup>[[User_Talk:Feba|OBJECTION!]]</sup> </small> 22:32, 28 May 2007 (UTC)


I saw that in the scientific literature, pseudouridine is considered the fifth nucleotide. For example:[https://pubmed.ncbi.nlm.nih.gov/27348156/], [https://pubmed.ncbi.nlm.nih.gov/37341888/]. My question is why is it called the fifth and not the sixth? To the best of my knowledge, when pseudouridine was discovered in the 1950's, the known nucleotides were: A,C,T,G,U = five nucleotides. So why isn't pseudouridine the sixth nucleotide? Thanks [[Special:Contributions/2A01:6500:A042:E52F:970A:37C0:5DE7:C30E|2A01:6500:A042:E52F:970A:37C0:5DE7:C30E]] ([[User talk:2A01:6500:A042:E52F:970A:37C0:5DE7:C30E|talk]]) 11:25, 21 May 2024 (UTC)
: i'm not sure about the training aspect but i doubt it. the basic principal behind glasses does NOT affect how ur eye works--rather it changes the incoming image so that it can be suitable for your eyes. take them away and you're back to the image your eyes can't properly focus.


:The original paper seems to be the 1956 publication [[doi:10.1016/S0021-9258(18)70770-9]] which, in Table 1, shows they only considered A,G,C and U as known in RNA at that time. [[User:Michael D. Turnbull|Mike Turnbull]] ([[User talk:Michael D. Turnbull|talk]]) 15:22, 21 May 2024 (UTC)
== What would happen if I were to eat lugduname? ==
:[[Thymine|T]] is not normally found in [[RNA]], being replaced by [[Uracil|U]] -- that must be the reason why. [[Special:Contributions/2601:646:8082:BA0:A400:D9D:C5FD:AB24|2601:646:8082:BA0:A400:D9D:C5FD:AB24]] ([[User talk:2601:646:8082:BA0:A400:D9D:C5FD:AB24|talk]]) 03:48, 24 May 2024 (UTC)


= May 22 =
What would happen if I were to eat [[Lugduname]]? would it burn a hole through my tongue? --[[User:Krsont|Krsont]] 22:29, 28 May 2007 (UTC)


== Net Ionic equation ==
== Evidence of physical empath ==


I have been looking for a reference for studies to find evidence of a physical empath, meaning a person who experiences physical pain that those around him or her are experiencing. I can find a plethora of web pages claiming empathy is real. I'm not looking for silly web pages. I'm trying to find scientific studies. So far, I only found ones about emotional empathy, not physical empathy. [[Special:Contributions/75.136.148.8|75.136.148.8]] ([[User talk:75.136.148.8|talk]]) 18:29, 22 May 2024 (UTC)
Is there any website where you do a net ionic equation?
:Here's something:
:[[Ionic equation]] has information. If you learn how, you can do a net ionic equation anywhere! On the bus, in a treehouse... [[User:Nimur|Nimur]] 00:47, 29 May 2007 (UTC)
:*{{cite web |last1=Armstrong |first1=Kim |title=‘I Feel Your Pain’: The Neuroscience of Empathy |url=https://www.psychologicalscience.org/observer/neuroscience-empathy |website=APS Observer |publisher=Association for Psychological Science |date=29 December 2017}}
:*{{cite journal |last1=Riess |first1=Helen |title=The Science of Empathy |journal=Journal of Patient Experience |date=June 2017 |volume=4 |issue=2 |pages=74–77 |doi=10.1177/2374373517699267 |url=https://www.ncbi.nlm.nih.gov/pmc/articles/PMC5513638/}}
:Unsure, however, if that satisfies your perception of "evidence of a physical empath". The neuroscience of empathy shows that observing others in pain can activate similar neural networks involved in experiencing pain firsthand.
::See also: [[Mirror-touch synesthesia]] --[[Special:Contributions/136.54.106.120|136.54.106.120]] ([[User talk:136.54.106.120|talk]]) 23:19, 22 May 2024 (UTC)
:::Thanks. I can get "I Feel Your Pain" from another local branch. That should work well. [[Special:Contributions/75.136.148.8|75.136.148.8]] ([[User talk:75.136.148.8|talk]]) 11:15, 23 May 2024 (UTC)


== Why does the northern hemisphere have two subtropical jets? ==
== Simple Static Mechanics Problem ==
I have 3 coins lying motionless on a frictionless surface. The 3 coins are all the same mass and size. The coins are arranged such that each coin is touching the other 2 coins. (the lines connecting their centers form an equilateral triangle) I apply a force F to a coin (#1) directly towards another coin (#2). In that instant, what are the forces on coins 1, 2 and 3 (the remaining coin)? [[User:Aepryus|Aepryus]] 00:00, 29 May 2007 (UTC)
:The point of contact will be a theoretical "single point" on each circular edge. The force will act along the normal to the mutual tangent. This will be the same as the line connecting the centers of each coin (if they are all the same size). Since these lines form an equilateral triangle, you will have 60-degree angles. Now you just need to assume that the force you apply is totally delivered to the opposite edge of the coin (no compression); a bit of trigonometry will tell you the numerical relations of the force. It will also depend on where you push the first coin (i.e. which point you are touching); in the ideal case, your force will always act normal to the tangent line of the circle. [[User:Nimur|Nimur]] 00:51, 29 May 2007 (UTC)
::Assuming [[static equilibrium]], the total force must sum to zero in each orthogonal direction.


There are normally two subtropical [[jet stream]]s in the winter northern hemisphere (e.g [https://www.meteo.physik.uni-muenchen.de/~roger/Lectures/TropicalMetweb/TropicalMeteorology_Ch1.html Fig12]), one over Africa-Asia-North Pacific and the other over North America-North Atlantic. Why is that so? Maybe the cold Sahara/[[Canary Current]] and [[California Current]] yield the gaps? [[User:JoJo Eumerus mobile|JoJo Eumerus mobile]] ([[User talk:Jo-Jo Eumerus|main talk]]) 18:55, 22 May 2024 (UTC)
= May 29 =
:Where do you see two subtropical jets? One of them is subtropical jet and the second is polar jet. [[User:Ruslik0|Ruslik]]_[[User Talk:Ruslik0|<span style="color:red">Zero</span>]] 19:13, 22 May 2024 (UTC)
::{{tq|Africa-Asia-North Pacific and the other over North America-North Atlantic}} They are not connected, and they both start in the subtropics. I am talking about east-west gaps, not north-south gaps. [[User:Jo-Jo Eumerus|Jo-Jo Eumerus]] ([[User talk:Jo-Jo Eumerus|talk]]) 07:07, 23 May 2024 (UTC)

= May 23 =

== Inner space ==

What is outer space "outer" of? Is it "the region beyond Earth's sky" (i.e. the atmosphere) mentioned in [[outer space#terminology]]? I tried visiting [[Inner space]], but it's a disambiguation page with no relevant results. [[User:Nyttend|Nyttend]] ([[User talk:Nyttend|talk]]) 02:14, 23 May 2024 (UTC)

:There isn't a firm boundary for outer space, as atmospheric pressure exponentially decreases with altitude above Earth's surface (see [[scale height]]) and thus the [[exosphere]] blends into space rather than suddenly vanishing into a vacuum. The lead section of [[outer space]] gives at least one boundary defined by convention, and the body states that {{tq|the density of atmospheric gas gradually decreases with distance from the object until it becomes indistinguishable from outer space}}; this need not occur at a fixed altitude even if we assume a constant pressure for the interplanetary medium (or threshold above it). I guess we could then say that outer space is "outer" to the region with a significantly higher density/pressure with respect to the interplanetary medium, or more simply, "outer" to any measurable atmosphere of a planet.
:I've also never heard the term "inner space" in the context of planetary science. <sup>[[User:ComplexRational|'''<span style="color:#0039a6">Complex</span>''']]</sup>/<sub>[[User talk:ComplexRational|'''<span style="color:#000000">Rational</span>''']]</sub> 02:38, 23 May 2024 (UTC)
: One arrives at inner space by sitting and meditating, not by visiting a website. <small>(Cue one of my frequent dad jokes: "My son has taken up meditation. Well, at least he's not just sitting around doing nothing".)</small> -- [[User:JackofOz|<span style="font-family: Papyrus;">Jack of Oz</span>]] [[User talk:JackofOz#top|<span style="font-size:85%; font-family: Verdana;"><sup>[pleasantries]</sup></span>]] 23:13, 23 May 2024 (UTC)

:[[Kármán line]] may be of interest. I don't think the term 'outer space' ever had a ''physical'' 'inner' counterpart. As a term for the mental realms, however, 'inner space' was often used in explicit opposition to 'outer space' in discussions of the '[[New Wave (science fiction)|New Wave]]' of Science Fiction writing in the 1960s and later (see [[Inner space (science fiction)]]). {The poster formerly known as 87.81.230.195} [[Special:Contributions/94.2.67.173|94.2.67.173]] ([[User talk:94.2.67.173|talk]]) 00:02, 24 May 2024 (UTC)
:Contrasting with outer space, there's also [[near space]], which redirects to mesosphere. [[User:PiusImpavidus|PiusImpavidus]] ([[User talk:PiusImpavidus|talk]]) 07:34, 24 May 2024 (UTC)

== What's the point of being enamored by one symmetric face and bored by an equally symmetric one? ==

Why should evolution give them such a wide range from meh to gobsmacking? [[User:Sagittarian Milky Way|Sagittarian Milky Way]] ([[User talk:Sagittarian Milky Way|talk]]) 05:16, 23 May 2024 (UTC)

:You are potentially ascribing too much intent, if rhetorical or metaphorical, to the process. Sometimes there are just [[spandrel (biology)|spandrel]]s. [[User:Remsense|<span style="border-radius:2px 0 0 2px;padding:3px;background:#1E816F;color:#fff">'''Remsense'''</span>]][[User talk:Remsense|<span lang="zh" style="border:1px solid #1E816F;border-radius:0 2px 2px 0;padding:1px 3px;color:#000">诉</span>]] 05:29, 23 May 2024 (UTC)
::If hunter gatherers lived in population densities under 1 per square mile and only saw one band of unrelated humans at a time and only about 75 were female and a fairly large percent were too young or too old with the stage 1 [[population pyramid]] and some of the rest aren't attracted to you and some you can't stand to live with long-term or almost and some can't stand you either then what's the point of some females being far more beautiful than others to specific men all other things being equal? Or it increased group harmony vs if beauty was less in the eye of the beholder? Less discord at least if not fights over women. [[User:Sagittarian Milky Way|Sagittarian Milky Way]] ([[User talk:Sagittarian Milky Way|talk]]) 00:43, 24 May 2024 (UTC)
:::This question doesn't seem like it has an answer, because you are ascribing too much intent to conceptual abstractions. [[User:Remsense|<span style="border-radius:2px 0 0 2px;padding:3px;background:#1E816F;color:#fff">'''Remsense'''</span>]][[User talk:Remsense|<span lang="zh" style="border:1px solid #1E816F;border-radius:0 2px 2px 0;padding:1px 3px;color:#000">诉</span>]] 01:04, 24 May 2024 (UTC)
:We have a [[facial symmetry]] article. [[User:DMacks|DMacks]] ([[User talk:DMacks|talk]]) 05:37, 23 May 2024 (UTC)
::It's nice that the last line in that article is the 'what the...?' cliffhanger "Some evidence suggests that face preferences in adults might be correlated to infections in childhood..." with the cited source saying "...frequency of diarrhea in particular". [[User:Sean.hoyland|Sean.hoyland]] ([[User talk:Sean.hoyland|talk]]) 06:40, 23 May 2024 (UTC)

* The word "bored" seems to have taken on some strange new meaning. I see people on social media saying stuff like "I was bored out of my skull so I came here ..." (= "I'm only here because I'm desperate"). If that isn't the greatest insult to their fellow socialmediaists, I don't know what would be. Contrary to the OP's title, being indifferent to something does not equate to being bored by it. One would have to spend some considerable time focussing on the object in question to get to the point of being bored by it, but that is the exact opposite of being so unimpressed by the object that one moves on immediately to something potentially more interesting. -- [[User:JackofOz|<span style="font-family: Papyrus;">Jack of Oz</span>]] [[User talk:JackofOz#top|<span style="font-size:85%; font-family: Verdana;"><sup>[pleasantries]</sup></span>]] 23:09, 23 May 2024 (UTC)
*:I think this sense of "bored" develops when one views a range of examples in aggregate such that they constitute one experience. [[User:Remsense|<span style="border-radius:2px 0 0 2px;padding:3px;background:#1E816F;color:#fff">'''Remsense'''</span>]][[User talk:Remsense|<span lang="zh" style="border:1px solid #1E816F;border-radius:0 2px 2px 0;padding:1px 3px;color:#000">诉</span>]] 23:17, 23 May 2024 (UTC)

= May 24 =

== Why is Aluminum so difficult to separate? ==

While the article on [[Aluminum]] talks about the fact that Aluminum wasn't really separated into pure or near-pure form until the 19th century, there doesn't seem to be an obvious reason given? Is it *somehow* tied to the other odd characteristic of Alumnimum, that when it oxidizes it goes the opposite way from Iron Oxide and flaking.[[User:Naraht|Naraht]] ([[User talk:Naraht|talk]]) 03:35, 24 May 2024 (UTC)
:This has to do with the fact that aluminum forms such a strong bond with [[oxygen]] that it can only be reduced to the metal by means of [[electrolysis]] or [[thermite process]] (plus the fact that, as also with [[zinc]] and a few other metals, the [[melting point]] of [[Aluminum oxide|the oxide]] is higher than the [[boiling point]] of the metal, so some way of reducing the former must be used to prevent the metal from simply boiling away). [[Special:Contributions/2601:646:8082:BA0:A400:D9D:C5FD:AB24|2601:646:8082:BA0:A400:D9D:C5FD:AB24]] ([[User talk:2601:646:8082:BA0:A400:D9D:C5FD:AB24|talk]]) 03:55, 24 May 2024 (UTC)
:Is it that odd to have a non-spalling oxide? In ultradry pure oxygen (partial pressure of H<sub>2</sub>O ≤2×10<sup>−5</sup> mmHg), even Li, Na, and K passivate (cite: Russell and Lee's book on nonferrous metals). :) [[User:Double sharp|Double sharp]] ([[User talk:Double sharp|talk]]) 12:51, 24 May 2024 (UTC)

::See: [[Hall–Héroult process]] -- Before then (1886), aluminum had been more expensive than gold.<sup>[https://gizmodo.com/how-aluminum-cost-more-than-gold-1575564897] [https://www.mgsrefining.com/blog/why-aluminum-is-no-longer-a-precious-metal/] [https://www.slate.com/articles/health_and_science/elements/features/2010/blogging_the_periodic_table/aluminum_it_used_to_be_more_precious_than_gold.html]</sup> --[[Special:Contributions/136.54.106.120|136.54.106.120]] ([[User talk:136.54.106.120|talk]]) 22:01, 24 May 2024 (UTC)

== [[Hexavalent chromium]] and [[heptavalent manganese]] ==

For the [[compound]]s with [[hexavalent chromium]] and [[heptavalent manganese]] (or the high [[oxidation state]]s of other [[transition metal]]s, such as hexavalent molybdenum, heptavalent technetium, hexavalent tungsten, heptavalent rhenium, etc.) and a single [[chemical element]], there are CrO3 and Mn2O7, but CrF6 and MnF7 seem to not exist, but are there CrS3, Mn2S7, CrCl6, MnCl7, etc.? Is O the only element which has compound with only two elements and with high [[oxidation state]]s of transition metals? +6 is one of the main oxidation state of chromium and +7 is one of the main oxidation state of manganese. [[Special:Contributions/61.224.168.169|61.224.168.169]] ([[User talk:61.224.168.169|talk]]) 10:29, 24 May 2024 (UTC)
:In general, really high oxidation states require O or F, essentially due to high electronegativity. Cr(VI) and Mn(VII) are high enough to be considered under this rubric. Which one is favoured depends on other factors, e.g. F has higher electronegativity but its monovalency instead of O's divalency leads to steric hindrance being more important. It can go either way: for Os, OsO<sub>4</sub> is well-known and OsF<sub>8</sub> nonexistent, but for Pt, it's PtF<sub>6</sub> that's well-known and PtO<sub>3</sub> that's not well-characterised.
:I don't think W(VI), for example, is really "high". Of course, it is the highest possible (just count valence electrons), but no one would call Na(I) high either. The early 4d and 5d metals tend to be happier in high oxidation states than their 3d counterparts. MoS<sub>3</sub>, MoCl<sub>6</sub>, WS<sub>3</sub>, WBr<sub>6</sub>, Tc<sub>2</sub>S<sub>7</sub>, and Re<sub>2</sub>S<sub>7</sub> all exist. And WF<sub>6</sub> is not even a great oxidising agent, whereas MnF<sub>4</sub> is already ferocious (and the highest fluoride, too). Late d-metals are indeed much more oxidising in their highest oxidation states: once you reach Ir and Pt, I would definitely agree that VI is high (PtF<sub>6</sub> famously oxidises dioxygen and xenon). But even then you see the pattern, since Co and Ni cannot even get as far as VI. [[User:Double sharp|Double sharp]] ([[User talk:Double sharp|talk]]) 12:49, 24 May 2024 (UTC)

== How do background gases influence combustion ==

If say, you ignite wood in a gas with the same [[partial pressure]] oxygen as air, but ten times the partial pressure of nitrogen. [[User:JoJo Eumerus mobile|JoJo Eumerus mobile]] ([[User talk:Jo-Jo Eumerus|main talk]]) 19:04, 24 May 2024 (UTC)
:Some gases like [[dibromotetrafluoroethane]] vapour can suppress burning. [[User:Graeme Bartlett|Graeme Bartlett]] ([[User talk:Graeme Bartlett|talk]]) 22:41, 24 May 2024 (UTC)
:The oxygen molecules move around randomly; to react with the fuel they need to bump into it. My guess is that the bump rate only depends on the partial pressure. &nbsp;--[[User talk:Lambiam#top|Lambiam]] 05:44, 25 May 2024 (UTC)
:An [[inert gas]] would still absorb some of the heat from the combustion, lowering the [[flame temperature]]. That should suppress the fire somewhat. Actual [[flame retardant]]s like abovementioned [[haloalkane]] aren't inert, but actively suppress the fire with the chemical reactions they undergo when heated or exposed to a flame. [[User:PiusImpavidus|PiusImpavidus]] ([[User talk:PiusImpavidus|talk]]) 10:21, 25 May 2024 (UTC)


= May 26 =

Latest revision as of 00:05, 26 May 2024

Welcome to the science section
of the Wikipedia reference desk.
Select a section:
Want a faster answer?

Main page: Help searching Wikipedia

   

How can I get my question answered?

  • Select the section of the desk that best fits the general topic of your question (see the navigation column to the right).
  • Post your question to only one section, providing a short header that gives the topic of your question.
  • Type '~~~~' (that is, four tilde characters) at the end – this signs and dates your contribution so we know who wrote what and when.
  • Don't post personal contact information – it will be removed. Any answers will be provided here.
  • Please be as specific as possible, and include all relevant context – the usefulness of answers may depend on the context.
  • Note:
    • We don't answer (and may remove) questions that require medical diagnosis or legal advice.
    • We don't answer requests for opinions, predictions or debate.
    • We don't do your homework for you, though we'll help you past the stuck point.
    • We don't conduct original research or provide a free source of ideas, but we'll help you find information you need.



How do I answer a question?

Main page: Wikipedia:Reference desk/Guidelines

  • The best answers address the question directly, and back up facts with wikilinks and links to sources. Do not edit others' comments and do not give any medical or legal advice.
See also:

May 12[edit]

ecological spray bottle[edit]

does anyone know if there are any glass or metal spray bottles with bioplastic triggers and straw available anywhere in existence? i really want to go plastic free for my succulent business ninosckasnaturals.com 2600:1700:9758:7D90:B406:C016:3BC0:D48B (talk) 06:05, 12 May 2024 (UTC)[reply]

Maybe one of those old-fashioned perfume misters with the rubber squeeze bulb? I doubt very much that there is a mass-produced non-plastic alternative spray bottle apparatus. Abductive (reasoning) 21:49, 12 May 2024 (UTC)[reply]
There are pump-type plant misters (e.g. metal or glass). --136.54.106.120 (talk) 18:27, 13 May 2024 (UTC)[reply]
P.s.: LOOPSEED sells stainless steel plant mister spray bottles in various finishes, well-suited for succulents (search online for details). --136.54.106.120 (talk) 18:49, 13 May 2024 (UTC) -- [edit: 22:33, 13 May 2024 (UTC)][reply]

origin of the formula for LC frequency[edit]

In electricity, properties known as inductance and capacitance together can resonate. The formula for the frequency of resonance is 1/(2*Pi*SQRT(L*C)). Who first published this formula? ```` Dionne Court (talk) 06:33, 12 May 2024 (UTC)[reply]

Somewhat after Laplace 1800 and before Poincarre, 1899 with a strong suspicion that the ubiquitous Maxwell might have done it. Greglocock (talk) 06:57, 12 May 2024 (UTC)[reply]
According to LC_circuit#History it was the ubiquitous Lord Kelvin in 1853. --Wrongfilter (talk) 07:10, 12 May 2024 (UTC)[reply]
Thank you. That man did everything. Greglocock (talk) 23:49, 12 May 2024 (UTC)[reply]
It does say that, but it is incorrect, which is why I posted here. Kelvin derived an equation to describe the transient response (response to a one-time shock excitation). However the article io LC_circuit#History gave as a reference an article in the Bell System Technical Journal, 1941, pages 415-453. I have now obtained this paper and it gives James Clerk Maxwell as the first to give the resonance formula (in a different but mathematically equivalent form), in a letter published in Philosphosical Magazine 1868. I will try and get this letter. ```` Dionne Court (talk) 03:13, 13 May 2024 (UTC)[reply]
It is this letter.  --Lambiam 12:14, 13 May 2024 (UTC)[reply]
It's from the right guy and via the correct other guy, but it has no math in it at all. It is not therefore the earliest statement of the resonance formula.
I'm looking for the fist statement of the formula as given in modern textbooks, i.e.,
f = 1/(2π(LC)½).
It is a trivial exercise in algrbra to convert Maxwell's form into the standard modern form, but I would like to know when the modern form was first give. Dionne Court (talk) 00:29, 14 May 2024 (UTC)[reply]
The section entitled Mathematical Theory of the Experiment, an enclosure to the letter immediately following it on page 361, definitely contains some maths. On page 363 we see the equation which results in an amplitude that, Maxwell writes, "is the greatest effect which can be produced with a given velocity". In this formula, the "velocity" is what is now more commonly denoted with the Greek letter  --Lambiam 06:49, 14 May 2024 (UTC)[reply]
p 540 onwards in Mathematical and Physical Papers, Volume 1 William Thomson Baron Kelvin University Press, 1882 - Mathematics - 619 pages, which is in google books, certainly discusses oscillatory behavior and time between peaks but I don't think it explicitly states f=1/(2pi*sqrt(L*C)). Particularly equation 7 where his A is modern L. Greglocock (talk) 00:08, 14 May 2024 (UTC)[reply]

May 14[edit]

Cranial size and Pb poisoning[edit]

Hello, can lead poisoning affect cranial size?Rich (talk) 05:43, 14 May 2024 (UTC)[reply]

Yes.[1] [2] [3] --136.54.106.120 (talk) 11:40, 14 May 2024 (UTC)[reply]

May 15[edit]

They see me rollin', they hatin...[edit]

Question for you guys. Is there any animal whose primary method of locomotion is curling into a ball and rolling head over heels to get around? Rather than running or walking. Because I think some woodlice do it (faster for them to roll then run), but I'm not 100% sure. Iloveparrots (talk) 03:04, 15 May 2024 (UTC)[reply]

Do fictional animals count? --136.54.106.120 (talk) 03:33, 15 May 2024 (UTC)[reply]
Terrestrial locomotion #Rolling may be of interest. --136.54.106.120 (talk) 03:38, 15 May 2024 (UTC)[reply]
I don't see how that could ever be a primary means of locomotion, but see also Category:Rolling animals. Shantavira|feed me 08:37, 15 May 2024 (UTC)[reply]
Hedgehogs do it to escape from predators when threatened. 2601:646:8082:BA0:BC05:6EA8:F933:9E6D (talk) 10:43, 15 May 2024 (UTC)[reply]
Hedgehogs roll up for protection, not for locomotion. (Ditto for the woodlice mentioned by the OP). Iapetus (talk) 11:57, 15 May 2024 (UTC)[reply]
I don't think I've ever seen a hedgehog roll outside of a video game. But then again, I've only ever seen hedgehogs in real life about three times. Iloveparrots (talk) 22:58, 15 May 2024 (UTC)[reply]
Hedgehogs do not purposely roll when in a defensive ball. They "huff", which makes them bounce, forcing their quills into whatever is attacking them. That bounce could cause a roll, but it isn't on purpose. 75.136.148.8 (talk) 13:40, 16 May 2024 (UTC)[reply]
See Rotating locomotion in living systems. 2605:B100:34D:46C3:61A4:6B17:A082:3780 (talk) 12:27, 15 May 2024 (UTC)[reply]
I have encountered assertions that Giant pandas, when sitting in a bamboo thicket on a slope, will sometimes roll a short distance rather than get up and walk, but this would need confirmation from a reliable source, and in any case would not be a primary means of locomotion. {The poster formerly known as 87.81.230.195} 94.2.67.173 (talk) 19:48, 15 May 2024 (UTC)[reply]
A giant panda is rolling head first in this video: Panda Discovers Something Interesting. They roll about a lot too, for various reasons. Modocc (talk) 23:30, 15 May 2024 (UTC)[reply]

What's the max depth of the Baltic, Black and Azov brim?[edit]

The depth where it stops being connected to the World Ocean 50% of the time (connected only by seepage through porous solids like silt not counting as connected). As the brim can erode, shift if the water's removed, be a V-notch in a ridge etc and even if it's dredged wide, straight and flat it might not be legal to lightly touch the silt so this might not round to the same number of feet as the deepest draft ship that's allowed at least 50% of the Metonic cycle, or how much sea level would have to drop to make it a lake 50% of the time. Also what would the depth be if the strait bottom wasn't landscaped? The Turkish Straits are pretty deep by ship standards maybe they aren't landscaped? Sagittarian Milky Way (talk) 18:05, 15 May 2024 (UTC)[reply]

They wouldn't need to be "landscaped", the Bosphorous is typically about 60 m deep with a 40 m sill towards its southern end, although the Asian side of the strait at that point is somewhat shallower (about 27 m) - see Siddall et al. (2004). Mikenorton (talk) 19:11, 15 May 2024 (UTC)[reply]
Right I didn't think they would. Unless the depth of the rim's now artificial by more than a foot due to sinking accidents(s), full or partial intentional blockages or explosion(s) (possibly to clear some of the previous)? Or maybe it's still the natural rock or sediment accumulation-erosion surface? I don't know if the Baltic and Azov brim are unaffected by human landscaping. Sagittarian Milky Way (talk) 20:16, 15 May 2024 (UTC)[reply]


May 17[edit]

What would cause these 'dark area' blemishes on an LCD monitor?[edit]

See this photo: https://ibb.co/mz8vQh0

This is my Asus Designo MX25AQ main monitor, I've had it for a little over four years and two months now. Since about a year ago, it started developing this issue where a "wavy" area of darkness appears near the left and right edges of the screen. It looks as if there's liquid in the screen or if something's "delaminating" inside. It would usually happen when it's cold, and would go away / "fade away" as it warmed up (usually taking a few minutes). However, over the last few months, it's been getting worse and worse. Nowadays, sometimes it's visible on certain shades of colours even when the monitor is fully warmed up. It is especially noticeable when it cools down in real life (e.g. it's becoming dawn and the brightness has been decreased).

The "grey uniformity" of the monitor has been degrading a bit as well, I swear.

I know I likely won't be able to fix an issue like this with the LCD panel. But my question is, what would possibly cause these issues? Could it develop into a stage where my LCD becomes completely / severely broken?

Note that I'm posting this to RD/S and not RD/C because I strongly believe this issue has something to do with materials degradation in the LCD and not some issue with the computer or cable, so I figured this is a better place for that.

Apologies for the non-free external image site upload, but if everything you see in the photo is not copyrighted then I will upload it to Commons. — AP 499D25 (talk) 05:03, 17 May 2024 (UTC)[reply]

I Googled around, and it could be moisture infiltrating around the edges. The fact that it clears when hot is suggestive of that. There was a suggestion that if the monitor is near a kitchen, these stains could include cooking fumes. Another possibility (and conceivably related) is damage from pulling the protective plastic sheet off, the one that came when the monitor was newly purchased. This has to be done extremely carefully. Yet another worry is cleaning with rubbing alcohol, the internet says this is a bad idea. Abductive (reasoning) 06:11, 17 May 2024 (UTC)[reply]
Rubbing alcohol as sold can consist for up to 50% of water.  --Lambiam 09:16, 18 May 2024 (UTC)[reply]
That sounds plausible! Thinking about it, more than a year ago I did clean my monitor using a wet microfibre cloth (a small, thin one for phones), and that seems to be the timepoint it all started. Worth pointing out that this monitor is in a bedroom, so it's not near moisture or oil vapour sources, and it has never been used in such an environment. When I got it new, there wasn't actually a protective sticky film on the front, just a styrofoam wrap. When I slid that wrap off, I got a nice big static shock when I touched the monitor bezel, but that didn't seem to do any immediate damage – this monitor was fine for the first 2.5 years or so that I used it.
Another interesting fact about this monitor is that although I bought it in Mar 2020, according to the info label, it has an manufacture date of Sep 2015, so that means it sat in a warehouse for 4.5 years before I bought it I guess.
Aside from this annoying and distracting issue, this monitor is the best quality display I've ever had (QHD resolution and 100% sRGB accuracy), so it'd be such a shame if it's actually dying on me. — AP 499D25 (talk) 08:06, 17 May 2024 (UTC)[reply]

Is there any scientific truth to the meme that Vegetable oil/Seed oil is toxic to the human body?[edit]

There are plenty of memes that seed oil causes diabetes and heart attacks but I failed to find any wikipedia articles that argues their case. Is there any scientific truth to the meme that Vegetable oil/Seed oil is toxic/harmful to the human body?

Another question is that if the meme of harmful seed oil is unscientific then why isn't this meme documented in the List_of_conspiracy_theories wiki page? 2001:8003:429D:4100:A593:8A5B:182E:5551 (talk) 15:55, 17 May 2024 (UTC)[reply]

Any kind of fats or oils in excess can cause heart disease, but there is no truth to the claim that vegetable oil is more toxic than animal fats (in fact, it's actually healthier) or that its consumption in moderation causes any health problems. 2601:646:8082:BA0:9480:50AE:ABF3:5E17 (talk) 23:30, 17 May 2024 (UTC)[reply]
Yes, there is truth to it. Our nutrition articles could do with some updating and balance. It is a prime example of Paracelsus's The dose makes the poison, for both the omega 6 LA and omega 3 ALA are essential nutrients for humans. The basic issue is consumption of a high quantity of omega-6 fatty acids from modern seed oils (which have only been consumed for a century or so) and other sources, including indirectly through animal feed and the (relative) paucity of omega-3 fatty acids in the modern human diet. [Excessive omega-3 over omega-6 has been found only in Greenlandic Inuit traditional communities.] Probably the best book to start with is Anthony John Hulbert's recent Omega Balance: Nutritional Power for a Happier, Healthier Life- Johns Hopkins (2022). By omega balance, he means the percentage of omega 3's in the sum of omega 3's and 6's. He says:

Although there is no advice about separate consumption of omega-3 and omega-6 fats in these national dietary guidelines, this is not the case with the premier scientific society concerning lipid research. In 2004, ISSFAL (International Society for the Study of Fatty Acids and Lipids) issued a series of recommendations for dietary intake of the essential fats by healthy adults. They made no comment about consumption of the nonessential saturated and monounsaturated fats but instead proposed that adequate intake of 18:2ω-6 [ Linoleic acid (LA) ] is 2 percent of energy, and a healthy intake of 18:3ω-3 [ α-linolenic acid (ALA) ] is 0.7 percent of energy as well as recommending a minimum intake of 500mg/d of 20:5ω-3 [ eicosapentaenoic acid (EPA)] and 22:6ω-6 [sic, should be 22:6ω-3 docosahexaenoic acid (DHA) as in the source [4]]. These ISSFAL recommendations for daily intake correspond to a diet omega balance of about 30 percent. The recommended intakes contrast markedly with the average actual daily intakes by the US population (from a 1999–2000 survey), which correspond to a diet omega balance of 9 percent. Similarly, a dietary survey of the Australian population revealed the average daily intake in 1995 corresponded to a diet omega balance of 11 percent. Both the United States and Australia (and likely many other developed high-income countries) have omega-6 intakes much higher and omega-3 intakes lower than the recommended levels.

Hulbert and other sources provide evidence that the omega imbalance can have deleterious effects not only in various chronic diseases, but also that the excess of inflammatory omega-6's can worsen outcomes of Covid, where many deaths appear to come from an excessive inflammatory response.John Z (talk)
μ-Oxidodihydrogen, a chemical compound found in industrially processed canned soup, is also known to be toxic to the human body. Why is no one talking about this?  --Lambiam 09:10, 18 May 2024 (UTC)[reply]
In very large excess, such as Olympic sized swimming pools, you would find it extremely difficult to swim in oil. Graeme Bartlett (talk) 12:37, 20 May 2024 (UTC)[reply]
It might be even more difficult in a bathtub.  --Lambiam 19:31, 20 May 2024 (UTC)[reply]
A wry comparison to the dangers of water is appropriate to much or most discussion of nutrition. Which proceeds by demonizing food X and then along with declaring it causes disease Y, X is declared to be a novelty even though it was consumed by all or much of humanity for tens of millennia at least, or is even essential to life. Historical absurdity is absurdly accepted. But it isn't relevant here because nobody considers modern seed oils to be anything but essentially new foods, never existing or consumed in such bulk before by humans or by any animal. And there aren't many such candidates for widespread dietary changes that could be implicated in global rises of chronic diseases.
Another source, which may conceal some such wryness, is Harumi Okuyama, Yuko Ichikawa, Yueji Sun, Tomohito Hamazaki, William Edward Mitchell Lands- Prevention of Coronary Heart Disease: From the Cholesterol Hypothesis to omega 6 omega 3 Balance- Karger (2006). They note that "Dietary advice was revealed to be the most serious risk factor for CHD in Japan." "We suggest that increased intake of LA [a consequence of that advice as it raised seed oil consumption] may be a major cause for the observed increase in CHD incidence in the group with dietary advice. Higher intakes of LA accompanied higher rates of CHD ( fig. 9–11 ; tables 4 , 5 ), whereas decreasing LA intake was effective for the secondary prevention of CHD events". In any case, there is a genuine, active scientific controversy here, not a conspiracy theory. Those who see omega imbalance as a real problem - and therefore seed oils, which uncontroversially are its ultimate source- may be the majority of specialist lipidologists, e.g. Artemis Simopoulos, cofounder of ISSFAL.John Z (talk) 04:20, 21 May 2024 (UTC)[reply]

Butterfly size[edit]

Do butterflies (especially nymphalids and/or swallowtails) become significantly smaller in size near the poleward (high-latitude) limit of their natural range? When I visited the Royal Ontario Museum in Toronto, they had 2 pinned specimens of Papilio cresphontes on display which were much smaller than their normal size per the article (one had a wingspan of "only" 3 inches -- I did a rough measurement with my fingers against the glass -- and the other was about 1/2 inch bigger) -- is this normal for (1) specifically P. cresphontes, (2) all swallowtails, and/or (3) all or most butterfly species? 2601:646:8082:BA0:9480:50AE:ABF3:5E17 (talk) 23:38, 17 May 2024 (UTC)[reply]

Size Distributions of Butterfly Species and the Effect of Latitude on Species Sizes (you can open a free JSTOR account or access through the Wikipedia Library). Alansplodge (talk) 10:10, 18 May 2024 (UTC)[reply]
Sorry, I won't click on any external links regarding this topic, just in case it might show me gratuitously enlarged pictures of P. multicaudata or some suchlike abomination -- can you just tell me the gist of it in a few words (or more than a few, your choice)? 2601:646:8082:BA0:E558:16C8:D2DE:51EF (talk) 10:31, 18 May 2024 (UTC)[reply]
There are no pictures, it's a scientific paper. "For butterfly species (Papilionoidea) of the Australian and Afrotropical regions, average wingspan decreases with increasing latitude". Alansplodge (talk) 10:34, 18 May 2024 (UTC)[reply]
Thanks! So the answer is yes -- right? (And that would explain the unusually small size of the two P. cresphontes specimens at the museum -- they must have been caught locally, and Toronto is near the northern limit of this species' natural range! And that is also quite reassuring for me -- it means that in Portland, Maine where I've been planning to move for quite a while, any P. cresphontes I come across won't be scary huge, in fact I might actually come to like them!) 2601:646:8082:BA0:E55E:2854:FEDE:FEB6 (talk) 21:21, 18 May 2024 (UTC)[reply]

May 18[edit]

Why packaging is important[edit]

Two reasons why packaging is important PhuPhumzile (talk) 07:10, 18 May 2024 (UTC)[reply]

Please do your own homework.
Welcome to the Wikipedia Reference Desk. Your question appears to be a homework question. I apologize if this is a misinterpretation, but it is our aim here not to do people's homework for them, but to merely aid them in doing it themselves. Letting someone else do your homework does not help you learn nearly as much as doing it yourself. Please attempt to solve the problem or answer the question yourself first. If you need help with a specific part of your homework, feel free to tell us where you are stuck and ask for help. If you need help grasping the concept of a problem, by all means let us know. Shantavira|feed me 09:16, 18 May 2024 (UTC)[reply]
Ya gotta love it when a poster doesn't even try to make it look like a question they thought of on their own. Or like a question, even. ←Baseball Bugs What's up, Doc? carrots→ 14:23, 18 May 2024 (UTC)[reply]
One reason: By packaging your homework question to make it look as if it is curiosity-driven, you have a much better chance of getting a useful answer.  --Lambiam 14:41, 18 May 2024 (UTC)[reply]
Why packaging is important: try buying loose helium by weight. Or by the handful. AndyTheGrump (talk) 15:30, 18 May 2024 (UTC)[reply]
In certain forms shrinkflation, despite selling less of a product, manufacturers keep the packaging at the original size and leave some of the space empty. This wrapping and transportation of air maintains the level of economic activity, which is important to the gross domestic product.  Card Zero  (talk) 21:22, 20 May 2024 (UTC)[reply]

May 19[edit]

Erythema Migrans[edit]

Hi

Does anyone have a good source for erythema migrans? I was looking for another one and can't copy (for my own notes) the Wikipedia page

W;ChangingUsername (talk) 19:04, 19 May 2024 (UTC)[reply]

Erythema migrans. ←Baseball Bugs What's up, Doc? carrots→ 20:19, 19 May 2024 (UTC)[reply]
I just came from there. The article isnt very good im sorry W;ChangingUsername (talk) 21:02, 19 May 2024 (UTC)[reply]
This Google Scholar search reduces the number of results from 19,400 to 11,900 by adding the term "review". Abductive (reasoning) 11:10, 20 May 2024 (UTC)[reply]
Thank you :)
I'll probably start using this for everything W;ChangingUsername (talk) 18:50, 20 May 2024 (UTC)[reply]

May 20[edit]

Can testosterone boost/etc... change someone desire from responsive to spontaneous?[edit]

Can testosterone boost/etc... change someone desire from responsive to spontaneous?177.207.104.19 (talk) 01:37, 20 May 2024 (UTC)[reply]

What do you mean? ←Baseball Bugs What's up, Doc? carrots→ 01:39, 20 May 2024 (UTC)[reply]
See Sexual desire and various other related articles, which you could have found easily by putting 'responsive desire' into the search box of this encyclopedia. You often post similar nonproductive responses to things which you personally have not heard of, although many others have: it becomes tedious. {The poster formerly known as 87.81.230.195} 94.2.67.173 (talk) 11:58, 20 May 2024 (UTC) [reply]
I see no harm in trying to encourage posters to link to what they're asking about. ←Baseball Bugs What's up, Doc? carrots→ 12:44, 20 May 2024 (UTC) [reply]
To be fair, there was a non-zero chance that it referred to a responsive to spontaneous change in the desire to fight strangers. Sean.hoyland (talk) 13:01, 20 May 2024 (UTC) [reply]
Theoretically. The funny thing is that the first sentence of 94.2.67.173's lecture to me could just as easily have served as a direct response to the poster. (Though maybe that was the point anyway!) ←Baseball Bugs What's up, Doc? carrots→ 14:40, 20 May 2024 (UTC) [reply]
You may be thinking of the DRD5 gene, where some alleles are thought to be involved in spontaneity or lack thereof. Abductive (reasoning) 11:15, 20 May 2024 (UTC)[reply]
Our article doesn't mention that, but implicates it in everything else: learning and memory, addiction, smoking, ADHD, Parkinson's disease, schizophrenia, locomotion, regulation of blood pressure, and immunity. What a busy gene. I guess you're referencing something along the lines of dopamine being important for the will to initiate movement, like in Awakenings.  Card Zero  (talk) 21:32, 20 May 2024 (UTC)[reply]
According to this review article, "of the few studies on T[estosterone] and desire in healthy women ... dyadic desire [desire in sex with a partner] has shown null or negative correlations with T". This suggests it is unlikely to achieve the specific effect.  --Lambiam 08:58, 21 May 2024 (UTC)[reply]

Has Gregory M. Cochran worked for Darpa? If so, in what capacity, if known?Rich (talk) 21:45, 20 May 2024 (UTC)[reply]

This Gregory Cochran would be a likely candidate, but I could find no direct evidence to support this. This Gregory M. Cochran seems less likely. There was a 'Doug Cochran' at DARPA, however. --136.54.106.120 (talk) 01:09, 21 May 2024 (UTC)[reply]
I bet those 2 Gregory Cochrans are the same.Rich (talk) 02:27, 21 May 2024 (UTC)[reply]
You're correct: they each are "co-author of the book The 10,000 Year Explosion." That Edge link could be used as a source for updating the article. --136.54.106.120 (talk) 05:57, 21 May 2024 (UTC)[reply]

May 21[edit]

Butterflies of Daviess County, KY (Papilionidae)[edit]

In Daviess County, Kentucky (or generally along the Ohio River or within a reasonable distance south thereof), how late in the year was the latest-ever sighting of Papilio glaucus? (Here in Central California, all tiger swallowtail species disappear in the first days of September -- my latest confirmed sighting of P. multicaudata was on September 1, and of P. rutulus on September 4 -- is it more-or-less the same over there?) Asking for a local and/or an expert -- and no pictures, please! 2601:646:8082:BA0:250E:98C8:7461:C819 (talk) 02:48, 21 May 2024 (UTC)[reply]

The pool of active RefDesk editors is quite small and is spread around the Anglosphere (or sometimes outside of it), so finding anyone from that locality or an entomologist here is a bit unlikely. A Google search only found backyardecology.net - Eastern Tiger Swallowtail Butterflies (with pictures) which says: "In Kentucky, we typically see the adults flying from April until September". Alansplodge (talk) 14:33, 21 May 2024 (UTC)[reply]
Thanks, but I was hoping for a bit more detail -- do they fly until early September (like their close relatives here in California), or until the end of September, or what? (The reason why I ask is, a friend of mine is over there on a farm caring for a sick relative, and I want to come over and help her with the farm itself, but at the same time I want to avoid any chance of a close encounter with one of those creepy critters out in the open (I think I told you more than once before how I feel about the Eastern tiger and about any other butterfly that looks like it -- with the exception of P. canadensis and P. machaon because they're nice and small, and also P. zelicaon because it's not only small but also its stripes are barely visible)! But I think since they only hatch in April (here in California I see P. rutulus beginning in mid-March), they shouldn't be flying any later in the year than they do here, so I should be "safe" beginning with the second week of September! (And just to be clear, other swallowtails like Papilio troilus are perfectly fine by me -- only tiger-striped ones aren't!) 2601:646:8082:BA0:A400:D9D:C5FD:AB24 (talk) 03:44, 24 May 2024 (UTC)[reply]
Broadening the topic somewhat, you might be interested (if you aren't already familiar with it) by the subject of Phenology. Climate change is obviously having a large influence on previously reliable annual timings of natural phenomena: in my part of the world (southern England), many trees are blooming, etc., up to a month earlier than a few decades ago. Doubtless the emergence, migrations and numbers of annual broods of insects are also changing, so they can appear later as well as earlier. {The poster formerly known as 87.81.230.195} 94.2.67.173 (talk) 09:57, 22 May 2024 (UTC)[reply]
Try calling 270-684-0211, the main number for the county public library in Owensboro. (Lest you embarrass yourself — be aware that the county name is pronounced "Davis", not "Davey's" or "Davy-ess".) Unfortunately their website's "contact" page, https://www.dcplibrary.org/contact, doesn't give either an email address or a form to write a help request. Nyttend (talk) 02:18, 23 May 2024 (UTC)[reply]
Thanks, but I just had a better idea -- is there a zoo and/or a university in Owensboro? Because if so, I think I'll try asking them (and if not, then I'll try Louisville -- the climate is more-or-less the same there, so these critters should disappear at about the same time in both places!) 2601:646:8082:BA0:C564:9993:1EA4:838E (talk) 23:17, 24 May 2024 (UTC)[reply]

Pseudouridine - why is it the fifth nucleotide?[edit]

I saw that in the scientific literature, pseudouridine is considered the fifth nucleotide. For example:[5], [6]. My question is why is it called the fifth and not the sixth? To the best of my knowledge, when pseudouridine was discovered in the 1950's, the known nucleotides were: A,C,T,G,U = five nucleotides. So why isn't pseudouridine the sixth nucleotide? Thanks 2A01:6500:A042:E52F:970A:37C0:5DE7:C30E (talk) 11:25, 21 May 2024 (UTC)[reply]

The original paper seems to be the 1956 publication doi:10.1016/S0021-9258(18)70770-9 which, in Table 1, shows they only considered A,G,C and U as known in RNA at that time. Mike Turnbull (talk) 15:22, 21 May 2024 (UTC)[reply]
T is not normally found in RNA, being replaced by U -- that must be the reason why. 2601:646:8082:BA0:A400:D9D:C5FD:AB24 (talk) 03:48, 24 May 2024 (UTC)[reply]

May 22[edit]

Evidence of physical empath[edit]

I have been looking for a reference for studies to find evidence of a physical empath, meaning a person who experiences physical pain that those around him or her are experiencing. I can find a plethora of web pages claiming empathy is real. I'm not looking for silly web pages. I'm trying to find scientific studies. So far, I only found ones about emotional empathy, not physical empathy. 75.136.148.8 (talk) 18:29, 22 May 2024 (UTC)[reply]

Here's something:
  • Armstrong, Kim (29 December 2017). "'I Feel Your Pain': The Neuroscience of Empathy". APS Observer. Association for Psychological Science.
  • Riess, Helen (June 2017). "The Science of Empathy". Journal of Patient Experience. 4 (2): 74–77. doi:10.1177/2374373517699267.
Unsure, however, if that satisfies your perception of "evidence of a physical empath". The neuroscience of empathy shows that observing others in pain can activate similar neural networks involved in experiencing pain firsthand.
See also: Mirror-touch synesthesia --136.54.106.120 (talk) 23:19, 22 May 2024 (UTC)[reply]
Thanks. I can get "I Feel Your Pain" from another local branch. That should work well. 75.136.148.8 (talk) 11:15, 23 May 2024 (UTC)[reply]

Why does the northern hemisphere have two subtropical jets?[edit]

There are normally two subtropical jet streams in the winter northern hemisphere (e.g Fig12), one over Africa-Asia-North Pacific and the other over North America-North Atlantic. Why is that so? Maybe the cold Sahara/Canary Current and California Current yield the gaps? JoJo Eumerus mobile (main talk) 18:55, 22 May 2024 (UTC)[reply]

Where do you see two subtropical jets? One of them is subtropical jet and the second is polar jet. Ruslik_Zero 19:13, 22 May 2024 (UTC)[reply]
Africa-Asia-North Pacific and the other over North America-North Atlantic They are not connected, and they both start in the subtropics. I am talking about east-west gaps, not north-south gaps. Jo-Jo Eumerus (talk) 07:07, 23 May 2024 (UTC)[reply]

May 23[edit]

Inner space[edit]

What is outer space "outer" of? Is it "the region beyond Earth's sky" (i.e. the atmosphere) mentioned in outer space#terminology? I tried visiting Inner space, but it's a disambiguation page with no relevant results. Nyttend (talk) 02:14, 23 May 2024 (UTC)[reply]

There isn't a firm boundary for outer space, as atmospheric pressure exponentially decreases with altitude above Earth's surface (see scale height) and thus the exosphere blends into space rather than suddenly vanishing into a vacuum. The lead section of outer space gives at least one boundary defined by convention, and the body states that the density of atmospheric gas gradually decreases with distance from the object until it becomes indistinguishable from outer space; this need not occur at a fixed altitude even if we assume a constant pressure for the interplanetary medium (or threshold above it). I guess we could then say that outer space is "outer" to the region with a significantly higher density/pressure with respect to the interplanetary medium, or more simply, "outer" to any measurable atmosphere of a planet.
I've also never heard the term "inner space" in the context of planetary science. Complex/Rational 02:38, 23 May 2024 (UTC)[reply]
One arrives at inner space by sitting and meditating, not by visiting a website. (Cue one of my frequent dad jokes: "My son has taken up meditation. Well, at least he's not just sitting around doing nothing".) -- Jack of Oz [pleasantries] 23:13, 23 May 2024 (UTC)[reply]
Kármán line may be of interest. I don't think the term 'outer space' ever had a physical 'inner' counterpart. As a term for the mental realms, however, 'inner space' was often used in explicit opposition to 'outer space' in discussions of the 'New Wave' of Science Fiction writing in the 1960s and later (see Inner space (science fiction)). {The poster formerly known as 87.81.230.195} 94.2.67.173 (talk) 00:02, 24 May 2024 (UTC)[reply]
Contrasting with outer space, there's also near space, which redirects to mesosphere. PiusImpavidus (talk) 07:34, 24 May 2024 (UTC)[reply]

What's the point of being enamored by one symmetric face and bored by an equally symmetric one?[edit]

Why should evolution give them such a wide range from meh to gobsmacking? Sagittarian Milky Way (talk) 05:16, 23 May 2024 (UTC)[reply]

You are potentially ascribing too much intent, if rhetorical or metaphorical, to the process. Sometimes there are just spandrels. Remsense 05:29, 23 May 2024 (UTC)[reply]
If hunter gatherers lived in population densities under 1 per square mile and only saw one band of unrelated humans at a time and only about 75 were female and a fairly large percent were too young or too old with the stage 1 population pyramid and some of the rest aren't attracted to you and some you can't stand to live with long-term or almost and some can't stand you either then what's the point of some females being far more beautiful than others to specific men all other things being equal? Or it increased group harmony vs if beauty was less in the eye of the beholder? Less discord at least if not fights over women. Sagittarian Milky Way (talk) 00:43, 24 May 2024 (UTC)[reply]
This question doesn't seem like it has an answer, because you are ascribing too much intent to conceptual abstractions. Remsense 01:04, 24 May 2024 (UTC)[reply]
We have a facial symmetry article. DMacks (talk) 05:37, 23 May 2024 (UTC)[reply]
It's nice that the last line in that article is the 'what the...?' cliffhanger "Some evidence suggests that face preferences in adults might be correlated to infections in childhood..." with the cited source saying "...frequency of diarrhea in particular". Sean.hoyland (talk) 06:40, 23 May 2024 (UTC)[reply]
  • The word "bored" seems to have taken on some strange new meaning. I see people on social media saying stuff like "I was bored out of my skull so I came here ..." (= "I'm only here because I'm desperate"). If that isn't the greatest insult to their fellow socialmediaists, I don't know what would be. Contrary to the OP's title, being indifferent to something does not equate to being bored by it. One would have to spend some considerable time focussing on the object in question to get to the point of being bored by it, but that is the exact opposite of being so unimpressed by the object that one moves on immediately to something potentially more interesting. -- Jack of Oz [pleasantries] 23:09, 23 May 2024 (UTC)[reply]
    I think this sense of "bored" develops when one views a range of examples in aggregate such that they constitute one experience. Remsense 23:17, 23 May 2024 (UTC)[reply]

May 24[edit]

Why is Aluminum so difficult to separate?[edit]

While the article on Aluminum talks about the fact that Aluminum wasn't really separated into pure or near-pure form until the 19th century, there doesn't seem to be an obvious reason given? Is it *somehow* tied to the other odd characteristic of Alumnimum, that when it oxidizes it goes the opposite way from Iron Oxide and flaking.Naraht (talk) 03:35, 24 May 2024 (UTC)[reply]

This has to do with the fact that aluminum forms such a strong bond with oxygen that it can only be reduced to the metal by means of electrolysis or thermite process (plus the fact that, as also with zinc and a few other metals, the melting point of the oxide is higher than the boiling point of the metal, so some way of reducing the former must be used to prevent the metal from simply boiling away). 2601:646:8082:BA0:A400:D9D:C5FD:AB24 (talk) 03:55, 24 May 2024 (UTC)[reply]
Is it that odd to have a non-spalling oxide? In ultradry pure oxygen (partial pressure of H2O ≤2×10−5 mmHg), even Li, Na, and K passivate (cite: Russell and Lee's book on nonferrous metals). :) Double sharp (talk) 12:51, 24 May 2024 (UTC)[reply]
See: Hall–Héroult process -- Before then (1886), aluminum had been more expensive than gold.[7] [8] [9] --136.54.106.120 (talk) 22:01, 24 May 2024 (UTC)[reply]

For the compounds with hexavalent chromium and heptavalent manganese (or the high oxidation states of other transition metals, such as hexavalent molybdenum, heptavalent technetium, hexavalent tungsten, heptavalent rhenium, etc.) and a single chemical element, there are CrO3 and Mn2O7, but CrF6 and MnF7 seem to not exist, but are there CrS3, Mn2S7, CrCl6, MnCl7, etc.? Is O the only element which has compound with only two elements and with high oxidation states of transition metals? +6 is one of the main oxidation state of chromium and +7 is one of the main oxidation state of manganese. 61.224.168.169 (talk) 10:29, 24 May 2024 (UTC)[reply]

In general, really high oxidation states require O or F, essentially due to high electronegativity. Cr(VI) and Mn(VII) are high enough to be considered under this rubric. Which one is favoured depends on other factors, e.g. F has higher electronegativity but its monovalency instead of O's divalency leads to steric hindrance being more important. It can go either way: for Os, OsO4 is well-known and OsF8 nonexistent, but for Pt, it's PtF6 that's well-known and PtO3 that's not well-characterised.
I don't think W(VI), for example, is really "high". Of course, it is the highest possible (just count valence electrons), but no one would call Na(I) high either. The early 4d and 5d metals tend to be happier in high oxidation states than their 3d counterparts. MoS3, MoCl6, WS3, WBr6, Tc2S7, and Re2S7 all exist. And WF6 is not even a great oxidising agent, whereas MnF4 is already ferocious (and the highest fluoride, too). Late d-metals are indeed much more oxidising in their highest oxidation states: once you reach Ir and Pt, I would definitely agree that VI is high (PtF6 famously oxidises dioxygen and xenon). But even then you see the pattern, since Co and Ni cannot even get as far as VI. Double sharp (talk) 12:49, 24 May 2024 (UTC)[reply]

How do background gases influence combustion[edit]

If say, you ignite wood in a gas with the same partial pressure oxygen as air, but ten times the partial pressure of nitrogen. JoJo Eumerus mobile (main talk) 19:04, 24 May 2024 (UTC)[reply]

Some gases like dibromotetrafluoroethane vapour can suppress burning. Graeme Bartlett (talk) 22:41, 24 May 2024 (UTC)[reply]
The oxygen molecules move around randomly; to react with the fuel they need to bump into it. My guess is that the bump rate only depends on the partial pressure.  --Lambiam 05:44, 25 May 2024 (UTC)[reply]
An inert gas would still absorb some of the heat from the combustion, lowering the flame temperature. That should suppress the fire somewhat. Actual flame retardants like abovementioned haloalkane aren't inert, but actively suppress the fire with the chemical reactions they undergo when heated or exposed to a flame. PiusImpavidus (talk) 10:21, 25 May 2024 (UTC)[reply]


May 26[edit]